Site Loader

Содержание

ЕГЭ по физике – силы трения. Отличия трения покоя и скольжения.

Трение на ЕГЭ по физике — каким оно бывает? Разберемся в этой короткой статье. А ещё расскажу, как связаны скрипы дверей и игра на скрипке.

 

Привет! Меня зовут Егор Блинов. Я – репетитор к ЕГЭ по физике. А ещё – преподаватель в МФТИ и преподаватель онлайн-школы Grand Exam.  В этой статье мы разберемся с тем, что такое сила трения и какие виды силы трения существуют. А главное – как их различать между собой.

Если не хочется читать – можно посмотреть материал в видео:

Сколько сил трения в ЕГЭ по физике? Явление застоя и скрипка!


Смотрите это видео на YouTube

Начнем разговор с того, что есть полная сила реакции опоры R.

 

R – это сила, с которой поверхность реагирует на тело, которое до неё дотрагивается.  Для бруска и для стержня, опирающихся на поверхность, сила полной реакции опоры выглядит так, как показано на рисунке. Вы заметили, что очень неудобно работать с такой силой. Потому что мы точно не знаем – куда она направлена. Что же мы сделаем? Мы разложим её на две перпендикулярных составляющих.

 

 

Одна из них будет – сила нормальной реакции опоры. Эта сила будет перпендикулярна поверхности. Еще будет сила трения – как раз о которой мы сегодня говорим. Сила трения – это составляющая силы R, которая направлена вдоль поверхности. Далее мы посмотрим, как между собой сила трения и сила N связаны. 

Разберемся с первым видом силы трения – силой трения покоя – на самом простом примере. Брусочек покоится на поверхности. Мы расставили силы нормальной реакции опоры: силы тяжести, сила F, с который мы тянем этот брусок, и сила трения, которая нам мешает сдвинуть этот брусок с места. Пока брусок покоится относительно поверхности, до того, как он начал движение, на брусок действует сила трения покоя. Если мы нарисуем график силы трения от силы F, которую мы прикладываем (тянем за веревочку), то он будет выглядеть вот так:

 

График похож на график прямой пропорциональности y=x из математики. То есть у этого графика тангенс угла наклона равен единице. Потому что x и y равны между собой. Если я увеличиваю силу, с которой я тяну брусок, то сила трения покоя тоже увеличивается и компенсирует наше дополнительное усилие. Поэтому наш грузик остается в покое.

Следующий случай – сила трения скольжения – это значит, что брусок начал скользить по поверхности. Сила трения перестает расти и достигает своего максимального значения

. – это некоторый коэффициент, о котором мы позже поговорим. N – это как раз сила нормальной реакции опоры. 

 

 

На графике видно, что до предела сила трения возрастала, а потом стала постоянной. Когда есть скольжение, и брусок движется относительно поверхности – сила трения скольжения, которая равна

. Примерно с постоянной скоростью я тяну грузик, и примерно одинаковой получается сила трения скольжения.

Теперь разберемся, что такое

. – это некоторый постоянный коэффициент, который будет характеризовать поверхности, которые трутся друг о друга. Например, для пары “металл-металл” он будет свой, для пары “металл-дерево” он будет уже другим. Он зависит только от материала и шероховатости трущихся поверхностей. В задачах ЕГЭ он не зависит ни от площади соприкосновения, ни от относительной скорости поверхностей.

В задачках

находится в пределах от нуля до единицы. Если ноль, значит вообще нет трения, если единица –   трения очень большое. В реальной жизни может быть и больше единицы – может быть, тело просто прилипло к поверхности.

Чтобы получить конспект всего ролика или конспекты по другим темам физики – просто напиши моему чат-боту слово конспект, и он тебе все пришлет.

Давайте все сведем в одну табличку и подытожим.

У нас есть силы трения покоя и скольжения. Основной критерий, по которому мы будем различать их: поверхности тела движутся относительно друг друга или еще нет. Если не движутся – значит, эта сила трения покоя; если движется – значит, сила трения скольжения. Как мы будем находить в задачах эти силы?

Если сила трения покоя – мы записываем Второй закон Ньютона и полагаем, что ускорение тела равна нулю (ведь оно покоится), и из Второго закона Ньютона можем найти эту силу трения. В случает трения покоя нельзя писать

, потому что формула  применяется, когда есть скольжение. Сила трения покоя ограничена величиной   – как вы помните из графика, сила трения покоя не может превышать этого значения. При этом направлена сила трения покоя будет против возможного движения тела. То есть против того направления, в котором двигалась бы тело, если бы силы трения не было.

Мы тянем грузик внешними силами F1 и F2  (это вид сверху), а сила трения препятствует. Так как это сила трения покоя, то грузик покоится, и из второго закона Ньютона сила трения равна сумме внешних сил. Но и поэтому она как раз и направлена противоположно тому направлению, в котором двигался этот грузик, если бы силы трения вообще не было бы.

А что по поводу силы трения скольжения – здесь уже имеет место движение. Модуль суммы сил F1 и F2  может быть больше, чем сила трения – тогда будет движение с ускорением.

Это основные моменты, которые мы должны знать, чтобы уметь различать эти силы.

Давайте пофантазируем, чем может быть полезно трение покоя и чем вредно. И то же самое для силы трения скольжения.

 

 

Например сила трения покоя полезна тем, что гвоздь не вылетает из отверстия, если мы его туда забили. Сила трения скольжения хороша тем, что можно на дороге затормозить “юзом” – если даже заблокировались колеса, все равно тормозим. Без силы трения скольжения было бы как-то страшно улететь в бесконечность.

Вредна сила трения покоя, потому что тяжело двигать мебель. А сила трения скольжения вредна тем, что происходит износ и нагрев подвижных деталей.

Напишите, пожалуйста, в комментарии, чем они полезны и чем вредны, и каким был бы наш мир, если бы не было никаких сил трения. Очень интересно узнать и дополнить свою презентацию вашими версиями.

А теперь давайте поговорим о явлении застоя. Обычно оно не учитывается в задачах ЕГЭ, и рассматривается “идеальный” график, хотя в реальности там может быть вот такой “клювик” – то есть превышение силы трения покоя над силой трения скольжения. Так мы определим явление застоя.

 

В видео мы можем обнаружить это явление экспериментально – пытаясь вывести грузик из состояния покоя, мы увеличиваем прикладываемую силу почти до 0.5 Н, но при движении, когда грузик срывается с места, устанавливается сила поменьше.  

Но самое необычное, что из-за этого явления у нас скрипит дверь,  и скрипка издаёт свой звук.
Наверное, вы замечали, что особенно сильно дверь скрипит, когда мы медленно её открываем. То есть сначала играет роль сила трения покоя, которая все больше увеличивается, а потом происходит “срыв” по клювику вниз, потом опять появляется какое-то движение, потом за счет силы трения это движение замедляется и прекращается, сила трения скольжения опять переходит в силу трения покоя, которая в свою очередь опять начинает нарастать.

И вот эти “срывы” заставляют дверь колебаться и издавать звук высокой частоты. Покой-движение-покой-движение – это чередование порождает звуковые колебания, которые нам обычно так противны. Но есть и плюс – благодаря этому явлению мы можем ходить на концерты классической
музыки со скрипкой.

Время экспериментов!

Поставим задачу: экспериментально определить коэффициент трения между кареткой (тележкой с грузиками) и деревянной поверхностью стола. Сначала найдем силу тяжести, которая действует на грузик:

 

 

Нарисовали рисунок, расставили силы, ввели оси координат, записали Второй закон Ньютона в проекции на обе оси. Если сила направлена вдоль оси – значит, учитываем её в уравнении с плюсом. Если против оси – значит, с минусом. Сила тяжести равна показаниям динамометра – 0,9Н.

Для случая, когда мы тащим грузик по поверхности, имеет место уже знакомая вам расстановка сил, мы рисовали её чуть раньше.

Запишем Второй закон Ньютона в проекциях на оси х и у. Получаем, что сила F, которую мы прикладываем к грузу, равна силе трения, которая рассчитывается по формуле.

На видео видно, что я примерно с постоянной скоростью тяну грузик, и примерно одинаковой получается сила трения скольжения.

Отсюда выражаем

 . Осталось подставить числа и подсчитать ответ:  .

До встречи на занятиях! Егор Блинов.

 

****************************************************

Ищу учеников на подготовку к ЕГЭ по физике. Индивидуальные дистанционные занятия и мини-группы. Подробная информация – в профиле.

А ещё на новогодних каникулах я буду проводить 6-дневный онлайн-интенсив по электродинамике на ЕГЭ. Длинные вебинары, домашка, доступ к консультациям до конца года. Цена при записи до НГ- 1490р. Записаться можно тут, а по промокоду КурсАгентЕгор можно получить скидку 200р.

****************************************************

Плюсы и минусы силы трения

Плюсы и минусы > Наука > Плюсы и минусы силы трения

В нашем мире все подчинено определенным законам. Любое действие, совершаемое на Земле, легко объясняется законами мироздания и природы. Что бы ни произошло, это легко объяснить, достаточно лишь немного освежить свои знания в области естествознания. Все законы физики, так или иначе, имеют свое влияние на происходящее в мире. Ну а если говорить о более насущных вещах, то даже на наш повседневный быт влияют законы физики. В особенности это касается силы трения. Что это за сила, и, чем она так знаменита? А главное, какую роль она оказывает на нашу повседневную жизнь?

Сила трения

Что такое сила терния? Сила трения – сила, возникающая при соприкосновении двух тел. Если объяснять это языком, далеким от науки, то, когда два предмета соприкасаются, то их поверхности редко бывают гладкими, поэтому на микроскопическом уровне их поверхности как бы цепляются друг за друга.

Плюсы

У этого природного явления есть много плюсов, которые делают нашу жизнь такой, какая она есть. Причем список этих плюсов достаточно внушительный:

  • Мы можем ходить. Если бы силы трения не существовало, то сложно представить, как бы мы перемещались. Наша стопа бы просто не могла бы сцепиться с землей, чтобы оттолкнуть тело в нужном направлении.
  • Мы можем стоять. Да-да, без силы трения мы не смогли бы ходить, но и стоять на месте тоже, любое дуновение ветерка могло бы «сдуть» нас куда угодно.
  • Мы можем носить в руках предметы. Все, что мы берем в руки не выскальзывает не только потому, что мы крепко держим предмет, а в основном благодаря силе трения.
  • Движение с помощью транспорта. Шины автомобилей могут отталкиваться от асфальта и двигать машину только благодаря силе трения. Поезд едет за счет сцепления с рельсами. Самокат, велосипед, ролики и другой транспорт с колесами был бы немыслим без силы трения.
  • Борьба с гололедом. По льду ходить затруднительно, а вот по льду, присыпанному песком – другое дело. Благодаря увеличению силы трения, мы можем перемещаться в пространстве даже зимой, когда дороги покрыты льдом.
  • Существование предметов. Все предметы соединены не только благодаря силе трения, но она играет очень важную роль. Даже нитки держат нашу одежду благодаря тому, что в физике есть такое явление.
  • Предметы могут тормозить. Яркий пример пользы этого явления — аварийные съезды. Во многих горных местностях есть специальные съезды на дорогах на случай, если у машины откажут тормоза. Достаточно поехать в гору некоторое время, тогда в дело вступит сила трения, и машина затормозит самостоятельно.
  • Предметы могут стоять. Представьте себе мир, где предметы могут только скользить и кататься. Наверно, это было бы чем-то похоже на космос и состояние невесомости. И попытка просто поставить на стол предмет оканчивалась бы провалом, мало того, что он выскальзывает из рук, так даже если бы это и удалось победить, то все равно, при попытке поставить стакан на стол, он бы просто скользил и падал.
  • Фрикционные механизмы. Их действие основывается как раз таки на силе трения. В отличие от зубчатых механизмов, фрикционные сцепляются за счет силы трения. И хотя они не так надежны, их применяют в областях, где важна бесшумность работы, например при изготовлении магнитофонов, проигрыва­телей, спидометров. Хотя нередко их можно встретить в различных станках, где важность имеют, прежде всего, точность регулирования.
  • Защита Земли от комет и метеоритов. За счет силы трения они сгорают еще до того, как успевают приблизиться к земле.

Минусы

Но даже у такой масштабной и важной природной силы есть свои минусы, которые немножко осложняют нам жизнь. Но их не так уж и много:

  • Движение тяжелых предметов. Чем меньше сила трения, тем легче сдвинуть предмет. Только вот в обычной жизни сила трения стандартная, что усложняет нам жизнь, когда нужно передвинуть какой-нибудь тяжелый предмет.
  • Предметы электризуются из-за силы трения. Конечно, в электризации предметов есть плюсы, но согласитесь, когда одежда бьется током и прилипает к телу, приятного в этом мало. Да и волосы, прилипающие к лицу и трещащие, когда пытаешься их пригладить.
  • Затрудняет работу различных механизмов за счет снижения коэффициента полезности действия. Для того чтобы увеличить КПД, приходится использовать различные вещества, которые помогают снизить силу трения.
  • Механизмы изнашиваются. Да и не только механизмы: подошва ваших любимых кед стирается, каменные ступеньки становятся скользкими, веревки перетираются, на носках появляются дырки – все это результат работы силы трения.
  • Невозможность создания вечного двигателя. Вечный двигатель – безумная мечта миллионов ученых за все время существования науки. Но недостижимая, потому что сила трения рано или поздно заставляет механизм остановиться.
  • Механизмы перегреваются
    . За счет силы трения возникает лишняя энергия, которая становится теплом, а затем нагревает элементы механизма. В некоторых случаях это может даже привести к возгоранию.
  • Спортивная скорость. Чтобы достигать высоких результатов, спортсмену необходимо напрячься, чтобы преодолеть силу трения. Многим спортсменам даже приходится брить свое тело, чтобы сделать кожу максимально гладкой. Как они утверждают, это помогает им снизить сопротивление воздуху, уменьшить силу трения и двигаться максимально быстро.

Вывод

Как можно увидеть, эта физическая сила – одна из важнейших вещей, которые существуют на нашей планете. Без этой физической величины наш мир был бы совершенно непохожим на тот, который мы знаем.

Да, несомненно, были бы какие-то мелкие плюсы, но минусов было бы намного больше, ведь в природе ничего не бывает просто так. Все продуманно и подчиняется определенным законам, которые создают для нас тот мир, который мы привыкли видеть.

XIII Международный конкурс научно-исследовательских и творческих работ учащихся Старт в науке

  • Авторы
  • Руководители
  • Файлы работы
  • Наградные документы

Ярина Т.М. 1


1МАОУ «Гимназия № 80 г. Челябинска»

Харитонова В.Е. 1Эсман Г.Е. 2


1МАОУ «Гимназия № 80 г. Челябинска»

2МБУДО «Центр детский экологический г. Челябинска»

Автор работы награжден дипломом победителя II степени

Диплом школьникаСвидетельство руководителяСвидетельство руководителя

Текст работы размещён без изображений и формул.
Полная версия работы доступна во вкладке «Файлы работы» в формате PDF

Введение

Я с первого класса увлекаюсь наукой физикой, посещаю внеурочные занятия, а также занимаюсь в экологическом центре.

Люблю изучать физические и биологические явления, проводить эксперименты. Уже ранее участвовала в научно-практических конференциях и в социальной программе «Шаг в будущее» с работами «Физика в игрушках», «Мои резинки и закон Гука», «Реактивное движение в природе и технике».

Но теперь, мне хочется изучить явление, без которого невозможна была бы наша жизнь на планете Земля. Это трение, и если бы оно исчезло, то мы бы скатились с нашей планеты, ведь она же вращается вокруг своей оси с большой скоростью, а для того, чтобы получить огонь, так необходимый для обогрева жилища, приготовления пищи, нужно было использовать явление трения, а значение трения в жизни растений и животных. Все это говорит об актуальности данной темы. Еще, очень люблю кататься на роликах, самокате, скейтборде, велосипеде, а для их движения, торможения большое значение имеет трение.

Цель моей работы: изучение явления трения, выяснения причины от чего зависит и не зависит сила трения, а также нахождение коэффициента трения.

Для достижения этой цели решались следующие задачи:

— подбор литературы;

— изучение, анализ литературы по данной теме;

— изучение пользы и вреда силы трения;

— изучение проявления силы трения в растительном и животном мире;

— проведение эксперимента по выяснению причин, от которых зависит сила трения и вычислению коэффициента трения.

Гипотеза исследования: сила трения скольжения зависит от силы движения, от рода соприкасающихся поверхностей, от площади поверхности.

В ходе работы использовались следующие методы исследования:

— теоретические (изучение, анализ и обобщение литературы по данной теме, наблюдения)

— экспериментальные (определение коэффициента трения, составления программы для его вычисления)

Практическая значимость работы состоит в том, что использование поставленных опытов, позволяет рассмотреть вопрос о величине силы трения скольжения более наглядно и учесть в быту влияние силы давления и качества обработки поверхностей на величину силы трения скольжения.

С таким явлением как трение человечество столкнулось и научилось его использовать на самой заре своего развития, предпринимая попытки добычи жизненно необходимого огня. Но и сегодня чем больше мы изучаем законы трения, тем сложнее, а не проще представляются они нам. Иными словами, чем глубже взгляд, чем аккуратнее измерения, тем сложнее становится истина. Тем больше у человека понимание того, что, расширяя круг знаний, он расширяет круг своих незнаний. Тем не менее, это не притупляет жажду его познания. Вот и мы в данной работе постараемся познакомиться с трением поближе и детально рассмотреть все его типы и формы, чтобы бороться с ним, когда оно нам мешает.
Трение принимает участие, и притом весьма не существенное, там, где мы о нем даже не подозреваем. Если бы трение внезапно исчезло бы из мира, множество обычных явлений протекало бы совершенно иным образом. Законы трения дают ответы на многие важные вопросы, связанные с движением тел. Мы видим, как разнообразно и порой неожиданно проявляется трение в окружающей нас обстановке.

Очень красочно пишет о роли трения французский физик Гийом: «Всем нам случалось выходить в гололедицу; сколько усилий стоило нам удержаться от падения, сколько смешных движений приходилось нам проделать, чтобы устоять! Это заставляет нас признать, что обычно земля, по которой мы ходим, обладает драгоценным свойством, благодаря которому мы сохраняем равновесие без особых усилий. Та же мысль возникает у нас, когда мы едем на велосипеде по скользкой мостовой или когда лошадь скользит по асфальту и падает. Изучая подобные явления, мы приходим к открытию тех следствий, к которым приводит трение. Инженеры стремятся по возможности устранить его в машинах и хорошо делают. Во всех прочих случаях мы должны быть благодарны трению: оно дает нам возможность ходить, сидеть и работать без опасения, что книги и чернильница упадут на пол, что стол будет скользить, пока не упрется в угол, а перо выскальзывать из пальцев.

Трение представляет настолько распространённое явление, что нам, за редкими исключениями, не приходится призывать его за помощью: оно является к нам само. Вообразим, что трение может быть устранено совершенно, тогда никакие тела, будь они величиною с каменную глыбу или малы, как песчинки, никогда не удержатся одно на другом: всё будет скользить и катиться, пока не окажется на одном уровне. Не будь трения, Земля представляла бы шар без неровностей, подобно жидкому.
К этому можно прибавить, что при отсутствии трения гвозди и винты выскальзывали бы из стен, ни одной вещи нельзя было бы удержать в руках, никакой вихрь никогда бы не прекращался, никакой звук не умолкал бы, а звучал бы бесконечным эхом, неослабно отражаясь, например, от стен комнаты.

Описать все проявления трения в нашей жизни не возможно, но мы смогли рассмотреть лишь некоторые из них и это очень увлекательно.

1. История изучения силы трения

Шёл 1500 год. Великий итальянский художник, скульптор и ученый Леонардо да Винче проводил странные опыты, чем удивил своих учеников.
Он таскал по полу, то плотно свитую веревку, то ту же веревку во всю длину. Его интересовал ответ на вопрос: зависит ли сила трения скольжения от величины площади соприкасающихся в движении тел? Механики того времени были глубоко убеждены, что чем больше площадь касания, тем больше сила трения. Они рассуждали примерно так, что чем больше таких точек, тем больше сила. Совершенно очевидно, что на большей поверхности будет больше таких точек касания, поэтому сила трения должна зависеть от площади трущихся тел.
Леонардо да Винчи усомнился и стал проводить опыты. И получил потрясающий вывод: сила трения скольжения не зависит от площади соприкасающихся тел. Попутно Леонардо да Винчи исследовал зависимость силы трения от материала, из которого изготовлены тела, от величины нагрузки на эти тела, от скорости скольжения и степени гладкости или шероховатости их поверхности. Он получил следующие результаты:
1.От площади не зависит

2. От материала зависит

3. От величины нагрузки зависит (пропорционально ей)

4. От скорости скольжения не зависит.

5. Зависит от шероховатости поверхности.

1699 год. Французский ученый Амонтон в результате своих опытов так ответил на те же пять вопросов. На первые три — так же, на четвертый – зависит. На пятый – не зависит. Получалось, и Амонтон подтвердил столь неожиданный вывод Леонардо да Винчи о независимости силы трения от площади соприкасающихся тел. Но в то же время он не согласился с ним в то, что сила трения не зависит от скорости скольжения; он считал, что сила трения скольжения зависит от скорости, а с тем, что сила трения зависит от шероховатостей поверхностей, не соглашался.
В течение восемнадцатого и девятнадцатого веков насчитывалось до тридцати исследований на эту тему. Их авторы соглашались только в одном – сила трения пропорциональна силе нормального давления, действующей на соприкасающиеся тела. А по остальным вопросам согласия не было. Продолжал вызывать недоумения даже у самых видных ученых экспериментальный факт: сила трения не зависит от площади трущихся тел. 1748 год. Действительный член Российской Академии наук Леонард Эйлер опубликовал свои ответы на пять вопросов о трении. На первые три – такие же, как и у предыдущих, но в четвертом он согласился с Амонтоном, а в пятом – с Леонардо да Винчи.
1779 год. В связи с внедрением машин и механизмов в производство назрела острая необходимость в более глубоком изучении законов трении. Выдающийся французский физик Кулон занялся решением задачи о трении и посвятил этому два года. Он ставил опыты на судостроительной верфи, в одном из портов Франции. Там он нашел те практические производственные условия, в которых сила трения играла очень важную роль. Кулон на все вопросы ответил – да. Общая сила трения в какой-то малой степени все же зависит от размеров поверхности трущихся тел, прямо пропорциональна силе нормального давления, зависит от материала соприкасающихся тел, зависит от скорости скольжения и от степени гладкости трущихся поверхностей. В дальнейшем ученых стал интересовать вопрос о влиянии смазки, и были выделены виды трения: жидкостное, чистое, сухое и граничное.

2. Понятие силы трения

Сила трения – это явление, возникающее при движении одного тела по поверхности другого и направленная в сторону, противоположную движению. Сила трения имеет электромагнитную природу.

Причины силы трения:

— шероховатость (неровность) поверхности;

-притяжение молекул.

Действие силы трения всегда сопровождается превращением механической энергии во внутреннюю энергию и вызывает нагревание тел и окружающей их среды.

Существует внешнее и внутреннее трение. Внешнее трение – вид трения, при котором в местах соприкосновения твёрдых тел возникают силы, затрудняющие взаимное перемещение тел и направленное по касательной к их поверхностям. Внутреннее трение (вязкость) – вид трения, состоящий в том, что при взаимном перемещении слоёв жидкости или газа между ними возникают касательные силы, препятствующие такому перемещению.

Внешнее трение подразделяют на трение покоя (статическое трение) и кинематическое трение.

Трение в общем виде — явление сопротивления относительному перемещению, возникающее между двумя телами в зоне соприкосновения их поверхностей. Оно характеризуется силой трения возникающей при всех видах трения, направленной вдоль поверхности соприкасающихся тел и препятствующих тел и препятствующей их относительному смещению. Трение подразделяется на трение покоя, трение скольжения и трение качения. (Приложение А)

Рассмотрим основные отличия.

Сила трения покоя возникает вдоль поверхности соприкосновения тел неподвижных относительно друг друга и препятствует возникновения движения одного тела относительно поверхности другого.

Сила трения скольжения возникает при относительном движении соприкасающихся тел и направлена против скорости их движения.
Сила трения качения возникает при качении одного тела по поверхности другого тела.

Данные о них представлены в таблице (Приложение Б)

Основной характеристикой трения является коэффициент трения (μ), который определяющийся отношением силы трения (Fтр) к силе давления (реакция опоры) (N): μ = FТр\N.

Теперь рассмотрим опыты, иллюстрирующие воздействие трения на предметы.

Потерев ластиком о поверхность стола, мы увидим, что на столе остались крошки от ластика и сделаем вывод, что трение приводит к износу трущихся поверхностей.

Потерев монетку о шершавую поверхность, мы увидим, что трение приводит к нагреву трущихся поверхностей. Эти проявления трения могут вызвать пожары.
Эти проявления трения нежелательны в технике, поэтому инженеры стараются всевозможными путями бороться с трением в узлах механизмов.
Так по возможности трение скольжения заменяют трение качения, применяя подшипники качения. Для снижения трения покоя и трения скольжения применяют смазку. Самым смелым решением борьбы с трением в технике является применение воздушных и магнитных подушек.

В тоже время без трения покоя между колесом и дорожным покрытием невозможно движение и в данном случае оно полезно. Интересно, что его необходимо увеличить при одновременном снижении трения качения колеса по дороге. И по сей день, инженеры находят все новые решения этой задачи, постоянно совершенствуя конструкцию и материалы современных шин.

3. Сила трения в царстве живой природы

Рассмотрим, зачем царствам живой природы необходимы силы трения.

3.1 Сила трения в жизни растений

• Усики многих растений имеют удобную форму для навивания и шероховатую поверхность для увеличения коэффициента трения.

• У растений, имеющих корнеплоды, такие, как морковь, свекла, брюква, сила трения о грунт способствует удержанию их в почве. С ростом корнеплода давление окружающей земли на него увеличивается, а это значит, что сила трения тоже возрастает. Поэтому так трудно вытащить из земли большую свеклу, редьку или репу.

• Семена же гороха, орехи благодаря своей шарообразной форме и малому трению качения перемещаются легко сами.

• Некоторые растения распространяют семена при помощи колючек:

репейник; чертополох.

3.2 Силы трения в жизни животных

• У многих растений и животных имеются различные органы, служащие для хватания

(хобот слона, цепкие хвосты лазающих животных). Все они имеют шероховатую поверхность для увеличения силы трения.

• На конечностях насекомых имеются шероховатые волоски для увеличения силы трения. Они помогают им передвигаться, удерживать пищу.

• У многих живых организмов существуют приспособления, благодаря которым трение получается небольшим при движении в одном направлении и резко увеличивается при движении в обратном направлении. Это, например, шерсть и чешуйки, растущие наклонно к поверхности кожи. На этом принципе основано Движение дождевого червя. Щетинки, направленные назад, свободно пропускают тело червя вперед, но тормозят обратное движение. При удлинении тела головная часть продвигается вперед, а хвостовая остается на месте, при сокращении головная часть задерживается, а хвостовая подтягивается к ней.

• Водяной жук – вертячка

• изумительно быстро носится на поверхности воды. Чтобы захватить их сачком, требуется большая ловкость. Вертячка — лучший пловец среди водных жуков.

• Оказывается, быстроте передвижения он во многом обязан покрывающей тело жировой смазке, которая значительно уменьшает трение о воду.

• К передвижению в водной среде животные приспособлены по-разному. Активные пловцы (рыбы, дельфины и др.) имеют характерную обтекаемую форму тела и конечности в виде плавников. Их быстрое плавание облегчается также особенностями строения внешних покровов и наличием специальной смазки ( слизи, снижающей трение о воду). Таким образом, «мягкость» или «волнистость» кожи дельфинов помогают им значительно уменьшать трение при скольжении в воде, а потеря частиц кожи по всему телу создает в процессе движения водовороты воды, которые сглаживают трение с потоком вокруг дельфина

• Обтекаемая форма тела для уменьшения сил сопротивления воздуха( птицы)

• При помощи клюва птицы добывают пищу, лазают по деревьям.

• В отсутствии трения покоя ни люди, ни животные не могли бы ходить по земле.

3.3 Силы трения в организме человека

• У животных и человека образующие сустав кости не касаются друг друга; они покрыты суставным хрящом, который выполняет роль буфера между костными поверхностями.

А по краям хряща прикрепляется синовиальная оболочка, в которой имеется жидкость, уменьшающая трение между суставными поверхностями.

• Сердце — полый мышечный орган, весом 250 — 300 гр., находится в околосердечной сумке из соединительной ткани. Внутренняя поверхность сумки выделяет жидкость, увлажняющую и уменьшающую трение при сокращении.

• Основной орган дыхательной системы – легкие, вокруг которых имеется плевральная жидкость.

• При глотании пищи и ее движении по пищеводу трение уменьшается за счет предварительного дробления и пережевывания пищи, а также смачивания ее слюной.

• Стопа ноги. Огромное преимущество нашей голой пятки и стопы в том, что они эластичны. Это дает им возможность «вписываться» в шероховатый рельеф, резко увеличивая площадь контакта с твердой поверхностью, а следовательно и сопротивление Особый мелкий гофр (волнистость) кожи на человеческой ступне также препятствует проскальзыванию ноги назад.

Выводы:

• Трение помогает растениям расти вверх, распространять семена;

• Птицы при помощи клюва и когтей добывают пищу;

• Хищники при помощи когтей преследуют, добывают пищу; устраивают норы;

• Рыбы могут уменьшать трение при помощи слизи, формы тела;

• Слаженная работа наших органов в организме.

• Хождение по земле

• Применение знаний силы трения в организме дает возможность в производстве бионики-техники ( самолеты….)

4. Времена года и трение

Рассмотрим, как зависит сила трения от погодных условий. Ведь это очень важно, люди ходят по поверхности, транспорт передвигается и в нем едут люди, и нам важно не получить травму и не попасть в аварию.

Лето.

Начало дождя – на грязном асфальте образуется грязевая пленка из пыли, песка, горюче смазочных материалов (вот почему асфальт иногда моют со специальными шампунями), а на раскалённом солнцем асфальте образуется еще и паровая пленка, это резко уменьшает трение, т. е. уменьшает сцепление автомобиля с дорожным полотном. А, значит, на полную остановку автомобиля уйдет намного больше времени и тормозной путь увеличится.

Ливень, дождь – вызывает серьезные проблемы: ухудшение видимости, уменьшение трения, скрытые лужами ямы, а так же вода, попадая в тормозные колодки, играет роль своеобразной смазки и сильно уменьшает эффективность работы тормозов.

Ребята, посмотрите в таблице, как и во сколько раз меняется коэффициент сцепления колес с дорогой, если дорога сырая. Как влияет дождь на дорожное покрытие? Рассмотрим таблицу (Приложение В)

— Да, трение уменьшается примерно в два раза, со всеми вытекающими отсюда последствиями!

Но есть еще более опасное явление, сводящее трение практически к нулю – гидропланирование (аква планирование). Сущность его состоит в том, что при достаточно высокой скорости и большой толщине водяной пленки в зоне контакта шин с дорогой появляется водяной клин, отрывающий колеса автомобиля от покрытия. Автомобиль как бы «приседает» на задних колесах, в то же время, как передние приподнимаются на водяном клине. Автомобиль перестает слушаться руля, хотя задние колеса продолжают сохранять сцепление с дорогой (вспомним, что большинство автомобилей у нас переднеприводные). По этой причине автомобиль даже на прямолинейных участках неожиданно оказывается на встречной полосе движения, а на закруглениях дорог внезапно съезжает на обочину или опрокидывается. Слой воды, толщиной несколько сантиметров может вызывать гидропланирование при скорости движения свыше 80 километров в час, особенно при изношенных шинах. Поэтому опытные водители при проезде участков, залитых водой, придерживаются скорости не выше 50-60 километров в час. Физически это явление основано на свойстве жидкости практически не сжиматься.

Весна, осень. Это очень опасный переходный период, когда погода резко меняется и меняется состояние дорожного полотна.

Заморозки – сильное уменьшение трения.

Ледяной дождь, вызывающий обледенение дорожного покрытия – трение для летней резины сводится практически к нулю.

Внезапный снегопад — ухудшение видимости, уменьшение трения.

Лиственная подушка на дороге (осенью) — уменьшение трения.

Перепад температуры через ноль – либо замерзание, либо таяние приводит к изменению сцепления, т.е. трение уменьшается в 8 раз.

Зимний период: Так же сложный период для вождения, не зря есть водители, которые вообще предпочитают зимой не ездить и держат свои автомобили до лета в гаражах.

Мокрый снег — ухудшение видимости, уменьшение трения особенно сказывается на склонах для тяжелых длинномерных автомобилей (фур).

Морозы – обледенение лобовых стекол и боковых зеркал.

А как меняется коэффициент трения на дороге покрытой снегом?

— уменьшается в 4 раза

Изучая все это, понимаем, что необходимо знать о силе трения, как можно больше, чтобы обезопасить свое передвижение на дорогах, как в качестве пешеходов, так и водителей.

Трение бывает сухое и вязкое, от чего зависит сила трения, как его уменьшить или увеличить при необходимости. Я поняла, что это очень важные вопросы.

ЭКСПЕРИМЕНТАЛЬНАЯ ЧАСТЬ РАБОТЫ:

Цель экспериментальной части: провести испытание вместе с папой, с нашей машиной и рассчитать коэффициент трения резины по разным дорогам.

Проезжая с папой на нашей машине часто замечала, как во время аварий, полицейские измеряют тормозной путь автомобиля, то есть бегают с рулетками.

Изучая силу трения и коэффициент трения, решила сама посмотреть, как вычислить коэффициент трения скольжения колес и от чего он зависит. Для этого я изучила учебник по физике за 9-й класс, научные книги по физике и потом формулы, по которым можно провести расчеты.

Коэффициент трения μ – это отношение Fтр к силе нормального давления (силе к весу тела ). Это формула тормозного пути автомобиля Sx=(vx2v0x2) / 2ax, начальная скоростьV0x, Vx— конечная скорость, она при торможении автомобиля Vx=0. ax – это ускорение – физическая величина, которая показывает, как быстро у автомобиля меняется скорость Sx=V0x2/2ax(чтобы найти ускорение, нужно V0x2/2Sx) следовательно, ax=V0x2/2Sx начальную скорость определяем так: смотрим на спидометр, когда нажимаем на тормоз. Sx– измеряя рулеткой в метрах.

Находим ускорение: Fтр= μN, где μ – коэффициент трения, N – сила опоры равная весу автомобиля N=P=mg, Fтр= μmg, где g – ускорение свободного падения на земле (g ≈ 9,8 м/с2 ≈ 10 м/с2)

С другой стороны Fтр=ma – этот закон еще открыл великий Ньютон.

μ mg=ma

μg=a

μ=a/g

Чтобы подсчитать коэффициент трения надо знать ускорение – aавтомобиля при торможении и g земли.

1.Автомобиль на ледяной дороге

Номер опыта

V0x, м/с

Начальная скорость (по спидометру)

Sx, м

Тормозной путь (измеряем рулеткой)

a=V0x²/2Sx, м/с2 – ускорение по формуле

μ =a/g

коэффициент трения (g=10 м/с2)

1.

10 м/с

10

5

0,5

2.

10 м/с

9.5

5.2

0,52

3.

10 м/с

9.7

5.15

0,515

Среднее значение

μ ср= (μ1+ μ2+ μ3)/ 3 = 1,535:3 = 0,511

2. Автомобиль на асфальте дороге

Номер опыта

V0x, м/с

Sx, м

a=V0х²/2Sx, м/с2

μ =a/g

1.

10 м/с

8,33

0,83

2.

10 м/с

6,2м

8,06

0,806

3.

10 м/с

5,8м

8,77

0,877

Среднее значение

μ ср= (μ1+ μ2+ μ3)/ 3 = 2,513:3 = 0,837

(подробнее см. приложение Г)

Выводы: коэффициент трения на обледенелой дороге намного меньше, чем на сухом асфальте.

В результате эксперимента, научилась рассчитывать коэффициент трения и поняла, что он не зависит от силы трения и силы давления, так как он равен отношению этих сил, а зависит от дорожного покрытия.

Провела много опытов, выяснила, от чего зависит и не зависит сила трения (убедилась, что Леонардо да Винчи во многом прав), что сила трения зависит от веса тела, рода трущихся поверхностей. Результаты четырех первых опытов размещены в приложении, а последняя практическая часть имела цель. (Приложение Д)

Заключение

Явление трения – это очень увлекательная тема для исследования. Трение надо знать и для объяснения механических явлений, тепловых явлений, биологических и даже астрономических. Трение связывает разные науки между собой, например, физику, биологию и астрономию. Я это поняла, изучая это явление. Провела много интересных опытов, выяснила причины, от чего зависит сила трения. Научилась вычислять коэффициент трения, скольжения разными методами.

В дальнейшем, я хочу подробнее изучить вязкое трение и понять, от чего оно зависит – это тоже важная задача, ведь многие животные, рыбы и даже человек плавают по воде, а также корабли, подводные лодки.

Список литературы:

Буров В.А., Кабанов С.Ф., Свиридов В.И. Фронтальные экспериментальные задания по физике в 6-7 классах средней школы : Пособие для учителей .-М.: Просвещение,2007.

Грабович В.Б. Папа Физика. – Челябинск: Издательство Игоря Розина, 2012.- 304с. УДК 53.02, ББК 74.262.22

Гуревич А. Е Физика. 7 класс. –М.: Дрофа, 2005.

Остер Г. Задачник по физике. -М.: АСТ: Астрель, 2001.

Перышкин А.В., Родина Н.А. Физика. Учебник для 7 класса. –М.: Народное образование,2006.

Робизин Ф.В.Простые опыты. Забавная физика для детей. — М.: Дет. лит., 2002 — 222стр.

Уроки физики с применением информационных технологий. 7-11 классы. Выпуск 2. Методическое пособие с электронным интерактивным приложением / З.В. Александрова, В.Н. Анатольев [и др.]; сост. З.В. Александрова. – М.: Планета, 2013. – 304 с. — (Современная школа).

Энциклопедический справочник школьника. ТОМ 1.Естественные науки-М.: Энциклопедическое товарищество, 2003год

Энциклопедия для детей. Том 16. Ч. 1.-М.: Аванта+, 2000

Приложение А

Понятие силы трения

Приложение Б

Трение.

Название силы

Формула

Направление

Условие применения

Примечание

Сила трения покоя

F< μ*N

Противоположно силе, приложенной к телу, вдоль поверхности соприкосновения.

Равна по величине и противоположна по направлению приложенной силы

Зависит от рода трущихся поверхностей и силы реакции опоры.
N – сила реакции опоры

Сила трения скольжения

F= μ*N

Противоположно направлению вектора относительной скорости движения.

Формула выполняется приближенно, т.к. сила трения зависит от скорости.

μ – коэффициент трения скольжения. Зависит от рода трущихся материалов, от обработки поверхности. Не зависит от площади соприкасающихся поверхностей.

Сила трения качения

F= μ*(N\R)

Противоположно направлению вектора относительной скорости движения.

Формула выполняется приближенно, т.к. сила трения зависит от скорости.

μ – коэффициент трения качения.

Приложение В

Плюсы и минусы трения

Положительные черты трения.

Представим себе на минуту, что сила трения внезапно исчезла. Окружающий нас мир стал бы совершенно иным. Мебель “гуляла бы” по комнате от легкого сквозняка, чашки и тарелки скользили бы по столу, с гор сползли бы все ледники, все камни и даже земля, лежащая на склонах. Даже самые спокойные из нас, учеников, не смогли бы усидеть за партами – при малейшем движении мы бы соскальзывали на пол. Все дома и другие конструкции рассыпались бы на составные части, так как все гвозди и шурупы выскочили бы. Поэтому трение покоя во многих случаях необходимо и выступает очень часто помощником человека.

Когда мы едем на автомобиле, нам необходимо остановиться. Ради этого водитель нажимает на тормоз. И тормозные колодки зажимают колесо. При этом происходит трение колеса о колодки и немного верхнего слоя шины стирается. Положительная роль трения еще и в том, что оно уменьшает тормозной путь автомобиля, что может спасти жизнь пешеходу при аварии. Если бы не трение, машина не сдвинулась бы с места. Зимой во время гололеда на автомобильных шинах устанавливают шипы для увеличения трения колес о дорогу.

Благодаря трению мы можем шкуркой шлифовать поверхности разных материалов и точить предметы даже из металла, т. к. проводя абразивной поверхностью шкурки по поверхности предмета, мы убираем неровности.

У многих растений и животных имеются различные органы, служащие для хватания (усики растений, хобот слона, цепкие хвосты лазающих животных). Все они имеют шероховатую поверхность для увеличения трения. Если не было бы трения, животные и растения не смогли бы хватать добычу.

Благодаря трению мы можем получать удовольствие от катания на роликах, велосипеде мопеде, мотоцикле. Ведь колёса этих средств передвижения крутятся, соприкасаясь с поверхностью земли, благодаря именно трению покоя и качения.

Чтобы добыть огонь с помощью двух, палочек, нужна большая сила трения.

Отрицательные черты трения.

Тяжелое тело бывает нелегко сдвинуть с места из-за силы трения.

Человечество уже много веков пытается создать “перпетум мобиле”, что в переводе с латинского обозначает “вечный двигатель”. Но из-за потерь энергии на преодоление трения, его создать невозможно.

Из-за силы трения может износиться деталь в швейной машинке (как и в любом другом механизме), а достать ее очень трудно. Поэтому соприкасающиеся детали механизмов делают гладкими, между ними вводят смазку. А в больших и тяжелых деталях машин используют подшипники скольжения или шариковые.

Сила трения портит экологию. Когда машина тормозит, те частицы, которые вырываются при трении шины колеса и асфальта уходят в воздух. Люди, животные, деревья, кусты их поглощают и становятся слабее, а некоторые даже умирают. Из-за трения возникают пожары на фабриках и заводах.

Сила трения мешает движению тел вперед. Из-за нее наши ботинки и туфли изнашиваются так быстро. Подошвы обуви касаются неровной поверхности земли и стираются.

Сила трения о воду мешает рыбам и дельфинам развивать большие скорости. Однако, дельфины способны плыть со скоростью автомобиля, хотя им приходится преодолевать сопротивление воды. При быстром плавании на коже животного возникает постоянная прослойка практически неподвижной воды. Эта прослойка называется ‛предельным слоем трения‛. В момент образования водных завихрений наделенная упругостью кожа начинает вибрировать. Кожа колеблется в обратном направлении от водных завихрений, которые образуются в предельном слое трения. Таким образом, гася их и препятствуя образованию трения воды. Результат: быстрое и свободное плавание. Такой принцип строения кожи дельфинов предлагается использовать в спортивных костюмах пловцов, в строительстве субмарин и кораблей.

Приложение Г

Дорога

Поверхность

Сухая

Мокрая

С асфальтобетонным или цементо -бетонным покрытием

0,7 — 0,8

0,35 — 0,45

С щебеночным покрытием

0,6 — 0,7

0,3 — 0,4

Грунтовая

0,5 — 0,6

0,2 — 0,4

Обледенелая

0,1 — 0,2

Покрытая снегом

0,2 — 0,3

Приложение Д

Сначала я стала выяснять, от чего зависит или не зависит Fтр.

Опыт № 1
Цель: проверить зависит ли сила трения от веса тела

Приборы: динамометр, брусок, грузы.

Ход работы: Я прикрепила динамометр к бруску и тянула по поверхности равномерно. Сила тяги равна по модулю силе трения. Потом, я подвесила брусок на динамометре и измерила вес бруска. Так я измерила силу трения. Потом, проделала тоже с бруском и одним грузом, двумя грузами, тремя грузами и внесла данные в таблицу.

Измеряем силу трения, двигая брусок равномерно с помощью динамометра по поверхности стола. Потом на брусок ставлю один груз. Подвешиваю к динамометру брусок 100 г. и снова двигаю, измеряю вес. Подвешиваю 2, потом 3 груза и измеряю вес.

Сделала вывод: Чем больше вес тела, тем больше сила трения при его движении. Также, я нашла коэффициент трения (разделила Fтр на вес P)

Номер опыта

Р, вес Н

FТрения , Н

Коэффициент трения μ

1

0.8

0.2

­0.25

2

1. 8

0.45

0.25

3

2.8

0.7

0.25

4

3.8

0.95

0.25

Вывод:

1)Чем больше вес тела, тем больше сила Трения

2)Коэффициент Трения не зависит от веса тела

Опыт № 2
Цель: Проверить зависит ли сила трения от рода трущихся поверхностей.
Приборы и материалы: брусок, грузики, динамометр.
Ход работы: Измеряю вес бруска и силу трения на разных поверхностях.

Номер опыта

Р, вес

FТрения

Коэффициент трения

μ=Fтр/P

1)дерево по дереву

0.8

0.4

0.5

2)дерево по песку

0. 8

0.5

0.6

3)дерево по стеклу

0.8

0.2

0.25

Вывод:Вывод: сила трения зависит от рода трущихся поверхностей. Коэффициент трения меняется самый большой на песке, самый маленький по стеклу (гладкая поверхность)

(см. видео фрагмент в презентации)

Опыт № 3
Цель: Проверить какая зависимость между P и Fтр
Приборы и материалы: брусок, грузики, динамометр.
Ход работы:

Номер опыта

Р, вес

FТрения

Вывод

1)брусок

0. 8

0.2

ВЕС ВСЕГДА БОЛЬШЕ ЧЕМ СИЛА ТРЕНИЯ

Между ними прямая зависимость

2)брусок 1 груза

1.8

0.45

 

3)брусок 2 груза

2.8

0.7

 

4)брусок 3 груза

3. 8

0.95

 

Опыт № 4

Цель: проверить зависит ли сила трения от площади трущихся поверхностей

Приборы: брусок и динамометр

Ход работы: Тянем равномерно брусок с помощью динамометра, меняя грани бруска

№ брусок

Fтр

Вывод

Большая грань

0,8

Сила трения не зависит от площади трущихся поверхностей

Средняя грань

0,8

Маленькая грань

0,8

Приложение Е

Хочу закончить свою работу стихотворением:

Трение – наш друг и враг.

Что такое трение?
Трение – явление.
Враг оно нам или друг?
Это знают все вокруг:
Если б трение пропало,
Чтоб со всеми нами стало?
Мы ходить бы не смогли,
Оттолкнувшись от земли.
Помогает трение.
Начинать движение.
Всем машинам, тракторам,
Мотоциклам, поездам.
Ну а также тормозить,
И их всех остановить.
Но при том приносит вред.
И немало разных бед:
В станках, приборах.
Трутся части —
И это главное несчастье.
И поэтому вопрос.
Не настолько уж и прост:
Тренье друг нам или враг?
Ответ двоякий – так и так!

Просмотров работы: 238

Сила трения это польза или вред

Конечно же, это фантазия, и она полна лирических упрощений. В жизни все немного по-другому. Но, по сути, несмотря на то, что есть очевидные минусы силы трения, которые создают для нас ряд сложностей в жизни, очевидно, что без существования сил трения, проблем было бы куда как побольше. Так что нужно говорить, как о вреде сил трения, так и о пользе все тех же сил трения.

Примерами полезных сторон сил трения можно назвать то, что мы можем ходить по земле, что наша одежда не разваливается, так как нитки в ткани удерживаются благодаря все тем же силам трения, что насыпав на обледеневшую дорогу песок, мы улучшаем сцепление с дорогой, дабы избежать аварии.

Ну а вредом силы трения является проблема перемещения больших грузов, проблема изнашивания трущихся поверхностей, а также невозможность создания вечного двигателя, так как из-за трения любое движение рано или поздно останавливается, требуя постоянного стороннего воздействия.

Люди научились приспосабливаться и уменьшать, либо увеличивать силы трения, в зависимости от необходимости. Это и колеса, и смазка, и заточка, и многое другое. Примеров масса, и очевидно, что нельзя однозначно сказать: трение – это хорошо или плохо. Но оно есть, и наша задача – научиться использовать его на пользу человека.

20

Трение в природе и технике

Пробовали ли вы ездить на автомобиле в гололед? Удовольствие не из приятных. Так же, впрочем, как и быть пешеходом в такую же пору года. Когда дорога покрыта коркой льда, мы говорим: плохое сцепление. Что это означает?

Это означает, что трение между колесами и дорогой очень маленькое. И если это полезно в случае перемещения грузов волоком, например, на санках, то очень вредно в ситуации, когда необходимо резко затормозить или сменить направление движения. Роль силы трения в жизни человека огромна, этого нельзя отрицать.

· И наша задача сводится к тому, чтобы максимально эффективно использовать силу трения в быту и в технике для облегчения жизни.

Роль силы трения в быту

Роль силы трения в быту сводится к тому, что мы можем ходить и ездить, что предметы не выскальзывают у нас из рук, что полки и картины висят на стенах, а не падают, даже одежду мы носим благодаря трению, которое удерживает волокна в составе нитей, а нити в структуре тканей.

Но трение может играть и отрицательную роль. Именно из-за него нагреваются и изнашиваются движущиеся части различных механизмов. В таких случаях его стараются уменьшить. Существует несколько способов уменьшения трения.

Один из них – это введение смазки между трущимися поверхностями. Смазка уменьшает соприкосновение тел, и трутся не тела, а слои жидкости. А трение в жидкости намного меньше, чем сухое трение.

Еще примеры силы трения в быту:

· мы можем писать на бумаге

· вещи, стоящие на вашем столе, не улетают от малейшего сквозняка

· одежда, которая висит на вашем стуле или плечиках в шкафу

· вы можете водите компьютерной мышкой по коврику

· вы с трудом двигаете шкаф, т.к. есть сила трения

· но если случайно разлить подсолнечное масло на кухне, любой входящий будет скользить, т.к. уменьшится сила трения об пол, но аккуратнее, не упадите сами 🙂

· ковер сильно уменьшает силу трения

· смазывание петлей дверей

· музыкальные инструменты

Сила трения в технике

Еще одним способом уменьшить трение является применение шариковых и роликовых подшипников. Внутреннее кольцо подшипника одевается на вал какого-либо механизма, а наружное кольцо закрепляют в корпусе машины или станка. И когда вал начинает вращаться, то он не скользит, а катится на шариках или роликах между кольцами подшипника.

А мы знаем, что сила трения качения значительно меньше трения скольжения. Поэтому вращающиеся части изнашиваются гораздо медленнее. Применяют также воздушную подушку, уменьшение площади соприкасающихся тел, а также шлифовку.

Например, чтобы уменьшить силу трения между льдом и коньками, коньки точат, делая поверхность соприкосновения меньше, а лед шлифуют, делая его максимально гладким. Так же уменьшают трение при резке чего-либо в быту и на производстве, затачивая ножи как можно острее.

Роль силы трения в технике не всегда отрицательна, как могло показаться. Ведь, например, когда мы заменяем силу трения скольжения трением качения, чтобы уменьшить взаимодействие трущихся поверхностей, то следует помнить, что если бы трение отсутствовало совсем, то колеса или шарики в подшипниках просто-напросто прокручивались бы, не приводя тело в движение.

Еще примеры силы трения в технике:

· автомобиль может тормозить

· на севере люди передвигаются на санках и лыжах — так быстрее, т.к. меньше сила трения

· езда на велосипеде

· любые смазанные детали работают лучше

· в шарикоподшипниках возникает сила трения качения

· колеса с шипами или даже с цепями

· механизмы для передачи или преобразования движения с помощью трения, т.н. фрикционные механизмы

Роль силы трения в природе

Стоит упомянуть и о роли силы трения в природе. Пример – это шероховатые лапки насекомых для улучшения сцепления с поверхностью, или, наоборот, это гладкие тела рыб, покрытые слизью для уменьшения трения о воду.

В природе животные и растения давно научились приспосабливаться и использовать силу трения себе во благо. То же необходимо делать и человеку, дабы обеспечить себе комфортное существование на планете Земля.

Еще примеры силы трения в природе:

· мы можем ходить по земле

· белки прыгают по веткам деревьев

· ленивец висит на ветке

· птичка может присесть на ветку

· вода точит камень

· образование планет и комет

· идет дождь и вода стекает в низину, хотя камень лежит и не скатывается в низину (у воды сила трения меньше, чем у камня)

· огромные валуны лежат на краях скал и не падают вниз — их держит сила трения

21

Презентация по физике по теме Сила трения.

Трение покоя. Трение в природе и технике доклад, проект
Слайд 1
Текст слайда:

Сила трения. Трение покоя. Трение в природе и технике.

д/з п. 30-32 закончить таблицу

Сила трения.
Обозначение и единицы измерения силы трения.
Виды силы трения.
Сравнение сил трения скольжения и трения качения.
Причины возникновения силы трения.
Способы уменьшения силы трения.


Слайд 2
Текст слайда:

Выяснить: а) причины возникновения
силы трения;
б) возможности ее
уменьшения;
в) факторы, от которых
зависит численное
значение силы трения.


Слайд 3
Текст слайда:

Fтр — cила трения — это сила, возникающая при движении одного тела по поверхности другого, и, направленная в сторону, противоположную движению.

Сила трения- сила, характеризующая взаимодействие, возникающее в месте соприкосновения тел и препятствующая их относительному движению.


Слайд 4
Текст слайда:

Обозначение: Fтр
Единица измерения: Н (Ньютон).


Слайд 5
Текст слайда:

Причины силы трения:
Шероховатость поверхностей соприкасающихся тел.
Неровности цепляются друг за друга и создают силу, задерживающую движение.

2. Взаимное притяжение молекул соприкасающихся тел.


Слайд 6
Текст слайда:

Сила трения скольжения- трение, которое возникает , если одно тело скользит по поверхности другого тела.

Сила трения качения -трение, которое возникает , если одно тело катится по поверхности другого тела

Сила трения покоя- трение, которое существует между покоящимися друг относительно друга телами, и, мешающая сдвинуть с места тело.

Виды силы трения:


Слайд 7
Текст слайда:

Сила трения покоя-
сила, препятствующая телам
приходить в движение

Сила трения покоя- трение, которое существует между покоящимися друг относительно друга телами, и, мешающая сдвинуть с места тело.


Слайд 8
Текст слайда:

Сила трения скольжения- это сила, возникающая при скольжении одного тела по поверхности другого

Сила трения скольжения- трение, которое возникает , если одно тело скользит по поверхности другого тела.


Слайд 9

Слайд 10
Текст слайда:

Виды силы трения:

Воздушное трение- трение, возникающее при движении в газе (воздухе)

Сухое трение- трение, возникающее при движении одного твердого тела по поверхности другого твердого тела

Жидкое трение- трение, возникающее при движении в жидкости или по поверхности жидкости


Слайд 11
Текст слайда:

Способы уменьшения трения:

1. Смазка трущихся поверхностей (трение между слоями жидкости слабее, чем между твердыми поверхностями)

2. Использование шариковых и роликовых подшипников

3. Применение воздушной подушки (трение уменьшается из-за создания воздушной области повышенного давления между телом и опорой. На этом принципе перемещаются суда на воздушной подушке.)


Слайд 12
Текст слайда:

Полезное трение

Благодаря трения покоя люди и животные ходят по земле.

Автомобили и поезда могут двинуться с места и остановиться.

Удерживает все стоящие на столе и на полу предметы.

Не будь трения, предметы выскальзывали бы из рук.


Слайд 13
Текст слайда:

Вредное трение

Нагреваются и изнашиваются многие движущиеся части различных механизмов.

Изнашиваются подошвы обуви и покрышки колёс автомобилей.


Слайд 14

Слайд 15

Слайд 16

Слайд 17

Слайд 18

Слайд 19

Слайд 20

Слайд 21
Текст слайда:

Трение в природе

У многих растений и животных  имеются различные органы, служащие для хватания (усики растений,  хобот слона, цепкие хвосты лазающих животных). Все они имеют шероховатую поверхность для увеличения силы трения. 


Слайд 22
Текст слайда:

Трение в природе

Среди живых организмов распространены приспособления (шерсть, щетина, чешуйки, шипы, расположенные наклонно к поверхности ), благодаря которым трение получается малым при движении в одном направлении и большим – при движении в противоположном направлении.


Слайд 23
Текст слайда:

Сила трения, действующая между двумя телами, неподвижными относительно друг друга, называют силой трения покоя.

Максимальное значение модуля силы трения покоя пропорционально модулю силы реакции опоры.


Слайд 24
Текст слайда:

 зависит от материала трущихся поверхностей, качества обработки

Максимальная сила трения покоя не зависит от площади соприкосновения тел.


Слайд 25

Слайд 26

Слайд 27

Слайд 28

Слайд 29

Слайд 30
Текст слайда:

Домашнее задание

Параграфы 30 — 32
Объясните эти поговорки о трении:
Не подмажешь -не поедешь
Пошло дело как по маслу
Угря в руках не удержишь
Что кругло- легко катится
Лыжи скользят по погоде
Из навощенной нити сеть не сплетёшь


Слайд 31
Текст слайда:

Решение задач.

Почему шины автомобилей делают с рифленой поверхностью?
Почему некоторые детали механизмов требуют смазки?
Приведите примеры, показывающие, что трение может быть полезным.
О какой опасности предупреждает дорожный знак с надписью «Осторожно, листопад!»?


Слайд 32
Текст слайда:

Тест 9А «Сила трения. Виды сил трения»
 


Слайд 33
Текст слайда:

Тест 9А «Сила трения. Виды сил трения»
 


Слайд 34
Текст слайда:

Тест 9А «Сила трения. Виды сил трения»
 

№3. Сила трения скольжения?
Сила, препятствующая началу движения
Сила, препятствующая качению
Сила, препятствующая скольжению
Сила, препятствующая движению тела в жидкости или газе


Слайд 35

Слайд 36
Текст слайда:

Тест 9А «Сила трения. Виды сил трения»
 


Слайд 37
Текст слайда:

Тест 9А «Сила трения. Виды сил трения»
 


Слайд 38
Текст слайда:

Тест 9А «Сила трения. Виды сил трения»
 


Слайд 39
Текст слайда:

Тест 9А «Сила трения. Виды сил трения»
 

№8. Формула для расчёта силы сухого трения?
F=mg
F= μ N=μmg
F=kx
F=S/t


Слайд 40

Слайд 41
Текст слайда:

Тест 9А «Сила трения. Виды сил трения»
 


Слайд 42
Текст слайда:

Тест 9А «Сила трения. Виды сил трения»
 


Слайд 43
Текст слайда:

Тест 9А «Сила трения. Виды сил трения»
 

№12. Какое трение позволяет удерживать лопату в руках?
Трение покоя
Трение скольжения
Трение качения


Слайд 44
Текст слайда:

Тест 9А «Сила трения. Виды сил трения»
 


Слайд 45
Текст слайда:

Тест 9А «Сила трения. Виды сил трения»
 


Слайд 46
Текст слайда:

Тест 9А «Сила трения. Виды сил трения»
 

№15. Что делают для уменьшения силы сухого трения?
Посыпают дороги песком
Надевают при работе с лопатой рукавицы на руки
Используют масло в двигателях
Используют подшипники
Зимой поливают снеговые горки водой


Слайд 47
Текст слайда:

Тест 9А «Сила трения. Виды сил трения»
 

№16. От чего зависит сила сухого трения?
От веса тела
От рода трущихся тел
От наличия неровностей на телах
От скорости движения тела
От площади соприкосновения трущихся тел


Слайд 48
Текст слайда:

Тест 9А «Сила трения. Виды сил трения»
 

№17. От чего зависит сила трения, действующая в жидкости или газе?
От веса тела
От скорости движения тела
От плотности жидкости или газа, в котором движется тело
От формы тела, обтекаемая или необтекаемая


Слайд 49
Текст слайда:

Тест 9А «Сила трения. Виды сил трения»
 

№18. Какие силы позволяют человеку стрелять из лука?
Сила тяжести
Сила упругости
Сила трения качения
Сила трения покоя
Сила трения скольжения


Слайд 50
Текст слайда:

Тест 9А «Сила трения. Виды сил трения»
 

№19. Почему плот, плавающий в воде, можно легко сдвинуть с места, а на земле это сделать очень трудно?
В жидкости сила трения качения меньше, чем на земле
В жидкости сила трения скольжения меньше, чем на земле
В жидкости нет трения покоя, а на земле есть
В воде действует жидкое трение, а на земле сухое, которое гораздо больше жидкого трения


Слайд 51
Текст слайда:

Тест 9А «Сила трения. Виды сил трения»
 

№20. Почему воздушный шар даже с мотором не может летать также быстро как самолёт?
Шар непрочный и не выдержит больших скоростей
Объём воздушного шара значительно больше, чем у самолёта, поэтому сила  воздушного трения будет гораздо больше


Слайд 52
Текст слайда:

Тест 9А «Сила трения. Виды сил трения»
 


Законы физики сила трения. Трение

Сила трения

Виды

При наличии относительного движения двух контактирующих тел силы трения, возникающие при их взаимодействии, можно подразделить на:

  • Трение скольжения — сила, возникающая при поступательном перемещении одного из контактирующих/взаимодействующих тел относительно другого и действующая на это тело в направлении, противоположном направлению скольжения.
  • Трение качения — момент сил , возникающий при качении одного из двух контактирующих/взаимодействующих тел относительно другого.
  • Трение покоя — сила, возникающая между двумя контактирующими телами и препятствующая возникновению относительного движения. Эту силу необходимо преодолеть для того, чтобы привести два контактирующих тела в движение друг относительно друга. Возникает при микроперемещениях (например, при деформации) контактирующих тел. Она действует в направлении, противоположном направлению возможного относительного движения.

В физике взаимодействия трение принято разделять на:

  • сухое , когда взаимодействующие твёрдые тела не разделены никакими дополнительными слоями/смазками (в том числе и твердыми смазочными материалами) — очень редко встречающийся на практике случай. Характерная отличительная черта сухого трения — наличие значительной силы трения покоя;
  • граничное , когда в области контакта могут содержаться слои и участки различной природы (окисные плёнки, жидкость и так далее) — наиболее распространённый случай при трении скольжения.
  • смешанное , когда область контакта содержит участки сухого и жидкостного трения;
  • жидкостное (вязкое) , при взаимодействии тел, разделённых слоем твёрдого тела (порошком графита), жидкости или газа (смазки) различной толщины — как правило, встречается при трении качения, когда твёрдые тела погружены в жидкость, величина вязкого трения характеризуется вязкостью среды;
  • эластогидродинамическое , когда решающее значение имеет внутреннее трение в смазывающем материале. Возникает при увеличении относительных скоростей перемещения.

В связи со сложностью физико-химических процессов, протекающих в зоне фрикционного взаимодействия, процессы трения принципиально не поддаются описанию с помощью методов классической механики .

Закон Амонтона — Кулона

Основной характеристикой трения является коэффициент трения , который определяется материалами, из которых изготовлены поверхности взаимодействующих тел.

В простейших случаях сила трения и нормальная нагрузка (или сила нормальной реакции) связаны неравенством

обращающимся в равенство только при наличии относительного движения. Это соотношение называется законом Амонтона — Кулона .

Закон Амонтона — Кулона с учетом адгезии

Для большинства пар материалов значение коэффициента трения не превышает 1 и находится в диапазоне 0,1 — 0,5. Если коэффициент трения превышает 1 , это означает, что между контактирующими телами имеется сила адгезии и формула расчета коэффициента трения меняется на

.

Прикладное значение

Трение в механизмах и машинах

В большинстве традиционных механизмов (ДВС , автомобили, зубчатые шестерни и пр.) трение играет отрицательную роль, уменьшая КПД механизма. Для уменьшения силы трения используются различные натуральные и синтетические масла и смазки. В современных механизмах для этой цели используется также напыление покрытий (тонких плёнок) на детали. С миниатюризацией механизмов и созданием микроэлектромеханических систем (МЭМС) и наноэлектромеханических систем (НЭМС) величина трения по сравнению с действующими в механизме силами увеличивается и становится весьма значительной , и при этом не может быть уменьшена с помощью обычных смазок, что вызывает значительный теоретический и практический интерес инженеров и учёных к данной области. Для решения проблемы трения создаются новые методы его снижения в рамках трибологии и науки о поверхности (англ. ).

Сцепление с поверхностью

Наличие трения обеспечивает возможность перемещаться по поверхности. Так, при ходьбе именно за счёт трения происходит сцепление подошвы с полом, в результате чего происходит отталкивание от пола и движение вперёд. Точно так же обеспечивается сцепление колёс автомобиля (мотоцикла) с поверхностью дороги. В частности, для увеличения улучшения этого сцепления разрабатываются новые формы и специальные типы резины для покрышек, а на гоночные болиды устанавливаются антикрылья , сильнее прижимающие машину к трассе.

См. также

Журналы

  • Трение, Износ, Смазка , журнал о трении.
  • Трение и Износ , журнал о трении издаётся Национальной Академией Наук Беларуси с 1980 г.
  • Journal of Tribology , международный журнал о трении.
  • Wear , международный журнал о трении и износе.
  • Таблицы коэффициентов трения , численные значения коэффициентов трения.

Литература

  • Дерягин Б. В. Что такое трение? М.: Изд. АН СССР, 1963.
  • Крагельский И. В., Щедров В. С. Развитие науки о трении. Сухое трение. М.: Изд. АН СССР, 1956.
  • Фролов, К. В. (ред.) Современная трибология: Итоги и перспективы . ЛКИ, 2008.
  • Bowden F. P., Tabor D. The Friction and Lubrication of Solids. Oxford University Press, 2001.
  • Persson Bo N. J.: Sliding Friction. Physical Principles and Applications. Springer, 2002.
  • Popov V. L. Kontaktmechanik und Reibung. Ein Lehr- und Anwendungsbuch von der Nanotribologie bis zur numerischen Simulation , Springer, 2009.
  • Rabinowicz E. Friction and Wear of Materials. Wiley-Interscience, 1995.

Ссылки

Wikimedia Foundation . 2010 .

Синонимы :

Смотреть что такое «Трение» в других словарях:

    Трение — – процесс, возникающий на поверхности соприкосновения тел, как находящихся в состоянии покоя, так и взаимного перемещения. … … Энциклопедия терминов, определений и пояснений строительных материалов

    Современная энциклопедия

    Трение — внешнее, механическое сопротивление, возникающее при относительном перемещении двух соприкасающихся тел в плоскости их касания. Сила сопротивления направлена противоположно относительному перемещению тел и называется силой трения. Трение… … Иллюстрированный энциклопедический словарь

    ТРЕНИЕ, противодействие перемещению соприкасающихся тел, направленное вдоль плоскости соприкосновения, а также противодействие жидкостям или газам, текущим по поверхности. Трение прямо пропорционально силе, сдавливающей поверхности, и зависит от… … Научно-технический энциклопедический словарь

    ТРЕНИЕ, трения, ср. 1. только ед. Состояние трущихся один о другой предметов, движение одного предмета по тесно соприкасающейся с ним поверхности другого. Машины изнашиваются от трения одних частей о другие. || Сопротивление движению, возникающее … Толковый словарь Ушакова

    ТРЕНИЕ, см. тереть. Толковый словарь Даля. В.И. Даль. 1863 1866 … Толковый словарь Даля

    ТРЕНИЕ, я, ср. 1. Сила, препятствующая движению одного тела по поверхности другого (спец.). Коэффициент трения. Кинематическое т. (между движущимися телами). Т. покоя (между неподвижными телами). 2. Движение предмета по тесно соприкасающейся с… … Толковый словарь Ожегова

    В аэро и гидродинамике касательные составляющие вектора поверхностных сил. Если в аэро и гидродинамических задачах движение жидкости или газа исследуется на основе Навье Стокса уравнений, то действие сил трения учитывается во всём поле течения, и … Энциклопедия техники

    Сопротивление движению, возникающее при перемещении соприкасающихся тел одно относительно другого. Различают Т. скольжения (Т. 1 го рода), появляющееся в результате скольжения одного тела по другому, и Т. качения (Т. 2 го рода), появляющееся в… … Морской словарь

Еще в школьные годы, в седьмом или восьмом классе, каждый человек знакомится с новым понятием динамической физики, — трением. Однако многие, повзрослев, забывают, и каким образом действует эта сила. Давайте попробуем разобраться в этой теме.

Определение понятия

Трение — это явление, которое заключает в себе следующий смысл: когда два тела соприкасаются друг с другом, на месте их контакта образуется особое взаимодействие, препятствующее телам продолжать движение относительно друг друга. Ясно, что можно подсчитать значение взаимодействия этих тел. как раз таки и характеризует данное взаимодействие количественно. Если трение происходит между твердыми телами (например, взаимодействие книги с книжной полкой или яблока со столом), то такое взаимодействие называется сухим трением.

Следует понимать, что трение — это сила, имеющая электромагнитную природу. Это означает, что причиной возникновения данной силы является взаимодействие между частицами, из которых состоит то или иное тело.

Каким бывает трение?

Благодаря разнообразию существующих в нашем мире предметов можно определить, что каждый из них имеет свою структуру и обладает индивидуальными свойствами. Это означает, что и взаимодействие между различными предметами будет отличаться. Для правильного понимания сути и грамотного решения многих задач в физике принято условно разделять три вида трения. Итак, разберем каждый по отдельности:

  • Первое трение — это трение покоя, которое возникает при отсутствии относительного перемещения двух тел. Мы можем наблюдать его примеры повсюду, ведь сила, возникающая при этом трении, удерживает предметы в равновесии. Например, товары на движущейся ленте транспортера, вбитый в стену гвоздь или человек, стоящий на полу.
  • Трение скольжения — это условно второе трение. Значение скольжения определяется таким образом: когда к телу, находящемуся в равновесии, прикладывают силу, которая больше, чем сила трения покоя, начинает действовать сила трения скольжения, и тело сдвигается с места.
  • И наконец, трение качения , объясняющее взаимодействие двух тел, одно из которых перекатывается по поверхности другого. Разница в и скольжения объясняется тем, что при любом движении площади тела смещаются по длине поверхности соприкосновения, и вместо разорванных межмолекулярных связей образуются новые. А в случае когда колесо катится без проскальзывания, молекулярные связи при подъеме участков колеса разрываются гораздо быстрее, чем при скольжении. Получается, что сила трения качения меньше силы скольжения.

Где и как можно использовать трение?

Трение — это незаменимое явление, без которого мы бы не смогли делать элементарные вещи: ходить, сидеть или же просто держать предметы в руках. Поэтому не стоит недооценивать значение трения. Как говорил французский физик Гильом: «Не будь трения, наша Земля была бы без единой шероховатости, она была бы подобна жидкой капле».

Пожалуй, лучший пример, который наиболее точно характеризует трение, — это работа колеса. Еще в древности было замечено, что силы трения качения гораздо меньше сил трения скольжения. Именно неоспоримая польза трения качения послужила причиной того, что люди стали подкладывать бревна или катки для перемещения тяжелых и габаритных грузов. С течением времени люди совершенствовали знания об удивительных свойствах трения качения, наблюдали за движением предметов под воздействием сил трения и, наконец, изобрели колесо! В современном мире невозможно представить жизни без этих незаменимых деталей, ведь колеса — это вторые «двигатели» любого транспорта!

Как вычислить значение силы трения?

Как и любая другая обладает целочисленными значениями. Для того чтобы точно определить, сколько силы потребуется для перемещения или других видов работ, необходимо подсчитать силу трения покоя. Этим обычно занимаются инженеры, когда, например, строят заводы или же изобретают новые устройства. Однако даже обычные школьники сталкиваются с определенными задачами, где требуется вычислить силу трения. Итак, чтобы подсчитать его значение, нужно просто воспользоваться несложной формулой: F трения = K * N, где k — это коэффициент трения. Значение всех коэффициентов зависит всегда от поверхности предмета, по которому движется или с которым взаимодействует тело. «N» в нашей формуле означает силу на тело. Она зависит в первую очередь от массы тела, которое соприкасается с поверхностью опоры.

Вычисляем значение силы в задаче

Допустим, тело массой m = 3 кг находится на горизонтальной доске. между деревянной доской и телом равен 0,3. Как же найти значение силы трения? Очень просто, всего-то нужно подставить наши значения в формулу. Только нужно учесть, что N в данном случае равен весу тела (по 3-му закону Ньютона). Итак, искомая сила равна (m * g) * k = (3 кг * 10 м/с 2) * 0,3 = 9 H.

«Физика — 10 класс»

Вспомните, что такое трение.
Какими факторами оно обусловлено?
Почему изменяется скорость движения по столу бруска после толчка?

Ещё один вид сил, с которыми имеют дело в механике, — это силы трения. Эти силы действуют вдоль поверхностей тел при их непосредственном соприкосновении.

Силы трения во всех случаях препятствуют относительному движению соприкасающихся тел. При некоторых условиях силы трения делают это движение невозможным. Однако они не только тормозят движение тел. В ряде практически важных случаев движение тела не могло бы возникнуть без действия сил трения.

Трение, возникающее при относительном перемещении соприкасающихся поверхностей твёрдых тел, называется сухим трением .

Различают три вида сухого трения: трение покоя, трение скольжения и трение качения.

Трение покоя.

Попробуйте сдвинуть пальцем лежащую на столе толстую книгу. Вы приложили к ней некоторую силу, направленную вдоль поверхности стола, а книга остаётся в покое. Следовательно, между книгой и поверхностью стола возникает сила, направленная против той силы, с которой вы действуете на книгу, и в точности равная ей по модулю. Это сила трения тp . Вы с большей силой толкаете книгу, но она по-прежнему остаётся на месте. Значит, и сила трения тp настолько же возрастает.

Силу трения, действующую между двумя телами, неподвижными относительно друг друга, называют силой трения покоя .

Если на тело действует сила , параллельная поверхности, на которой оно находится, и тело при этом остаётся неподвижным, то это означает, что на него действует сила трения покоя тp , равная по модулю и направленная в противоположную сторону силе (рис. 3.22). Следовательно, сила трения покоя определяется действующей на него силой:

Если действующая на покоящееся тело сила хотя бы немного превысит максимальную силу трения покоя, то тело начнёт скользить.

Наибольшее значение силы трения, при котором скольжение ещё не наступает, называется максимальной силой трения покоя .

Для определения максимальной силы трения покоя существует весьма простой, но не очень точный количественный закон. Пусть на столе находится брусок с прикреплённым к нему динамометром. Проведём первый опыт. Потянем за кольцо динамометра и определим максимальную силу трения покоя. На брусок действуют сила тяжести m, сила нормальной реакции опоры 1 , сила натяжения 1 , пружины динамометра и максимальная сила трения покоя тр1 (рис. 3.23).

Положим на брусок ещё один такой же брусок. Сила давления брусков на стол увеличится в 2 раза. Согласно третьему закону Ньютона сила нормальной реакции опоры 2 также увеличится в 2 раза. Если мы снова измерим максимальную силу трения покоя, то увидим, что она увеличилась во столько раз, во сколько раз увеличилась сила 2 , т. е. в 2 раза.

Продолжая увеличивать число брусков и измеряя каждый раз максимальную силу трения покоя, мы убедимся в том, что

>максимальное значение модуля силы трения покоя пропорционально модулю силы нормальной реакции опоры.

Если обозначить модуль максимальной силы трения покоя через F тр. mах, то можно записать:

F тр. mах = μN (3.11)

где μ — коэффициент пропорциональности, называемый коэффициентом трения. Коэффициент трения характеризует обе трущиеся поверхности и зависит не только от материала этих поверхностей, но и от качества их обработки. Коэффициент трения определяется экспериментально.

Эту зависимость впервые установил французский физик Ш. Кулон.

Если положить брусок на меньшую грань, то F тр. mах не изменится.

Максимальная сила трения покоя не зависит от площади соприкосновения тел.

Сила трения покоя изменяется в пределах от нуля до максимального значения, равного μN. За счёт чего может происходить изменение силы трения?

Дело здесь вот в чём. При действии на тело некоторой силы оно слегка (незаметно для глаза) смещается, и это смещение продолжается до тех пор, пока микроскопические шероховатости поверхностей не расположатся относительно друг друга так, что, зацепляясь одна за другую, они приведут к появлению силы, уравновешивающей силу . При увеличении силы тело опять чуть-чуть сдвинется так, что мельчайшие неровности поверхностей по-иному будут цепляться друг за друга, и сила трения возрастёт.

И лишь при > F тр. mах ни при каком взаимном расположении шероховатостей поверхности сила трения не в состоянии уравновесить силу , и начнётся скольжение.

Зависимость модуля силы трения скольжения от модуля действующей силы показана на рисунке 3.24.

При ходьбе и беге на подошвы ног действует сила трения покоя, если только ноги не скользят. Такая же сила действует на ведущие колёса автомобиля. На ведомые колёса также действует сила трения покоя, но уже тормозящая движение, причём эта сила значительно меньше силы, действующей на ведущие колёса (иначе автомобиль не смог бы тронуться с места).

В давнее время сомневались, что паровоз сможет ехать по гладким рельсам. Думали, что трение, тормозящее ведомые колёса, будет равно силе трения, действующей на ведущие колёса. Предлагали даже делать ведущие колёса зубчатыми и прокладывать для них специальные зубчатые рельсы.

Трение скольжения.

При скольжении сила трения зависит не только от состояния трущихся поверхностей, но и от относительной скорости движения тел, причём эта зависимость от скорости является довольно сложной. Опыт показывает, что часто (хотя и не всегда) в самом начале скольжения, когда относительная скорость ещё мала, сила трения становится несколько меньше максимальной силы трения покоя. Лишь затем, по мере увеличения скорости, она растёт и начинает превосходить F тр. mах.

Вы, вероятно, замечали, что тяжёлый предмет, например ящик, трудно сдвинуть с места, а потом двигать его становится легче. Это как раз и объясняется уменьшением силы трения при появлении скольжения с малой скоростью (см. рис. 3.24).

При не слишком больших относительных скоростях движения сила трения скольжения мало отличается от максимальной силы трения покоя. Поэтому приближённо можно считать её постоянной и равной максимальной силе трения покоя:

F тр ≈ F тр. mах = μN.

Силу трения скольжения можно уменьшить во много раз с помощью смазки — чаще всего тонкого слоя жидкости (обычно того или иного сорта минерального масла) — между трущимися поверхностями.

Ни одна современная машина, например двигатель автомобиля или трактора, не может работать без смазки. Специальная система смазки предусматривается при конструировании всех машин.

Трение между слоями жидкости, прилегающими к твёрдым поверхностям, значительно меньше, чем между сухими поверхностями.

Трение качения.

Сила трения качения существенно меньше силы трения скольжения, поэтому гораздо легче перекатывать тяжёлый предмет, чем двигать его.

Сила трения зависит от относительной скорости движения тел. В этом её главное отличие от сил тяготения и упругости, зависящих только от расстояний.

Силы сопротивления при движении твёрдых тел в жидкостях и газах.

При движении твёрдого тела в жидкости или газе на него действует сила сопротивления среды. Эта сила направлена против скорости тела относительно среды и тормозит движение.

Главная особенность силы сопротивления состоит в том, что она появляется только при наличии относительного движения тела и окружающей среды.
Сила трения покоя в жидкостях и газах полностью отсутствует.

Это приводит к тому что усилием рук можно сдвинуть тяжёлое тело, например плавающую лодку, в то время как сдвинуть с места, скажем, поезд усилием рук просто невозможно.

Модуль силы сопротивления F c зависит от размеров, формы и состояния поверхности тела, свойств среды (жидкости или газа), в которой тело движется, и, наконец, от относительной скорости движения тела и среды.

Примерный характер зависимости модуля силы сопротивления от модуля относительной скорости тела показан на рисунке 3.25. При относительной скорости, равной нулю, сила сопротивления не действует на тело (F c = 0). С увеличением относительной скорости сила сопротивления сначала растёт медленно, а затем всё быстрее и быстрее. При малых скоростях движения силу сопротивления можно считать прямо пропорциональной скорости движения тела относительно среды:

F c = k 1 υ, (3.12)

где k 1 — коэффициент сопротивления, зависящий от формы, размеров, состояния поверхности тела и свойств среды — её вязкости. Вычислить коэффициент k 1 теоретически для тел сколько-нибудь сложной формы не представляется возможным, его определяют опытным путём.

При больших скоростях относительного движения сила сопротивления пропорциональна квадрату скорости:

F c = k 2 υ 2 , υ, (3.13)

где k 2 — коэффициент сопротивления, отличный от k 1 .

Какую из формул — (3 12) или (3.13) — можно использовать в конкретном случае, определяется опытным путём. Например, для легкового автомобиля первую формулу желательно применять приблизительно при 60-80 км/ч, при больших скоростях следует использовать вторую формулу.

Инструкция

Пример задачи 3: брусок массой 1 кг соскользнул с вершины наклонной плоскости за 5 секунд, путь 10 метров. 2 = 0,8 м/с2. Вычислите силу трения в первом случае: Fтр = 1*9,8*sin(45о)-1*0,8 = 7,53 Н. Определите силу трения во втором случае: Fтр = 1*9,8*sin(45о)+2-1*0,8= 9,53 Н.

Случай 6. Тело двигается по наклонной поверхности равномерно. Значит, по второму закону Ньютона система находится в равновесии. Если скольжение самопроизвольное, движение тела подчиняется уравнению: mg*sinα = Fтр.

Если же к телу приложена дополнительная сила (F), препятствующая равноускоренному перемещению, выражение для движения имеет вид: mg*sinα–Fтр-F = 0. Отсюда найдите силу трения: Fтр = mg*sinα-F.

Источники:

  • скольжение формула

При относительном движении двух тел между ними возникает трение. Оно также может возникнуть при движении в газообразной или жидкой среде. Трение может как мешать, так и способствовать нормальному движению. В результате этого явления на взаимодействующие тела действует сила .

Инструкция

Наиболее общий случай рассматривает силу , когда одно из тел закреплено и покоится, а другое скользит по его поверхности. Со стороны тела, по которому скользит движущееся тело, на последнее действует сила реакции опоры, направленная перпендикулярно плоскости скольжения. Эта сила буквой N.Тело может также и покоится относительно закрепленного тела. Тогда сила трения , действующая на него Fтртрения . Он зависит от материалов трущихся поверхностей, степени их отшлифовки и ряда других факторов.

В случае движения тела относительно поверхности закрепленного тела сила трения скольжения становится равна произведения коэффициента трения на силу реакции опоры: Fтр = ?N.

Пусть теперь на тело действует постоянная сила F>Fтр = ?N, параллельная поверхности соприкасающихся тел. При скольжении тела, результирующая составляющая силы в горизонтальном направлении будет равна F-Fтр. Тогда по второму закону Ньютона, ускорение тела будет связано с результирующей силой по формуле: a = (F-Fтр)/m. Отсюда, Fтр = F-ma. Ускорение тела можно найти из кинематических соображений.

Часто рассматриваемый частный случай силы трения при соскальзывании тела с закрепленной плоскости. Пусть? — угол наклона плоскости и пусть тело соскальзывает равномерно, то есть без . Тогда уравнения движения тела будут выглядеть так: N = mg*cos?, mg*sin? = Fтр = ?N. Тогда из первого уравнения движения силу трения можно выразить как Fтр = ?mg*cos?.Если тело движется по наклонной плоскости с a, то второе уравнение будет иметь вид: mg*sin?-Fтр = ma. Тогда Fтр = mg*sin?-ma.

Видео по теме

Если сила, направленная параллельно поверхности, на которой стоит тело, превышает силу трения покоя, то начнется движение. Оно будет продолжаться до тех пор, пока движущая сила будет превышать силу трения скольжения, зависящую от коэффициента трения. Рассчитать этот коэффициент можно самостоятельно.

Вам понадобится

  • Динамометр, весы, транспортир или угломер

Инструкция

Найдите массу тела в килограммах и установите его на ровную поверхность. Присоедините к нему динамометр, и начинайте двигать тело. Делайте это таким образом, чтобы показатели динамометра стабилизировались, поддерживая постоянную скорость . В этом случае сила тяги, измеренная динамометром, будет равна с одной стороны силе тяги, которую показывает динамометр, а с другой стороны силе , умноженной на скольжения.

Сделанные измерения позволят найти данный коэффициент из уравнения. Для этого поделите силу тяги на массу тела и число 9,81 (ускорение свободного падения) μ=F/(m g). Полученный коэффициент будет один и тот же для всех поверхностей такого же типа, как и те на которых производилось измерение. Например, если тело из двигалось по деревянной доске, то этот результат будет справедлив для всех деревянных тел, двигающихся скольжением по дереву, с учетом качества его обработки (если поверхности шершавые, значение коэффициента трения скольжения измениться).

Можно измерить коэффициент трения скольжения и другим способом. Для этого установите тело на плоскости, которая может менять свой угол относительно горизонта. Это может быть обыкновенная дощечка. Затем начинайте аккуратно ее за один край. В тот момент, когда тело придет в движение, скатываясь в плоскости как сани с горки, найдите угол ее уклона относительно горизонта. Важно, чтобы тело при этом не двигалось с ускорением. В этом случае, измеренный угол будет предельно малым, при котором тело начнет двигаться под . Коэффициент трения скольжения будет равен тангенсу этого угла μ=tg(α).

Видео по теме

Сила реакции опоры относится к силам упругости, и всегда направлена перпендикулярно поверхности. Она противостоит любой силе, которая заставляет тело двигаться перпендикулярно опоре. Для того чтобы рассчитать ее нужно выявить и узнать числовое значение всех сил, которые действуют на тело, стоящее на опоре.

Вам понадобится

  • — весы;
  • — спидометр или радар;
  • — угломер.

Инструкция

Определите массу тела с помощью весов или любым другим способом. Если тело находится на горизонтальной поверхности (причем неважно, движется оно или пребывает в состоянии покоя), то сила опоры равна силе тяжести на тело. Для того чтобы рассчитать ее умножьте массу тела на ускорение свободного падения, которое равно 9,81 м/с² N=m g.

Когда тело движется по наклонной плоскости, направленной под углом к горизонту, сила реакции опоры находится под углом в силе тяжести. При этом она компенсирует только ту составляющую силы тяжести, которая действует перпендикулярно наклонной плоскости. Для расчета силы реакции опоры, с помощью угломера измерьте угол, под которым плоскость располагается к горизонту. Рассчитайте силу реакции опоры, перемножив массу тела на ускорение свободного падения и косинус угла, под которым плоскость находится к горизонту N=m g Cos(α).

В том случае, если тело движется по поверхности, которая представляет собой часть окружности с радиусом R, например, мост, то сила реакции опоры учитывает силу, по направлению из центра окружности, с ускорением, равным центростремительному, действующую на тело. Чтобы рассчитать силу реакции опоры в верхней точке, от ускорения свободного падения отнимите квадрата скорости к радиусу .

Получившееся число умножьте на массу движущегося тела N=m (g-v²/R). Скорость должна быть измерена в метрах в секунду, а радиус в метрах. При определенной скорости значение ускорения, направленного от центра окружности, может сравняться, и даже ускорение свободного падения, в этот момент сцепление тела с поверхностью пропадет, поэтому, например, автомобилистам, нужно четко контролировать скорость на таких участках дороги.

Если же направлена вниз, и траектория тела вогнутая, то рассчитайте силу реакции опоры, прибавив к ускорению свободного падения отношение квадрата скорости и радиуса кривизны траектории, а получившийся результат умножьте на массу тела N=m (g+v²/R).

Источники:

  • сила опору

Движение в реальных условиях не может продолжаться до бесконечности. Причина этому – трения . Она возникает при контакте тела с другими телами и всегда направлена противоположно направлению движения. Это означает, что сила трения всегда выполняет отрицательную работу , что нужно учитывать при расчетах.

Вам понадобится

  • — рулетка или дальномер;
  • — таблица ля определения коэффициента трения;
  • — понятие о кинетической энергии;
  • — весы;
  • — калькулятор.

Инструкция

Если тело движется равномерно и прямолинейно, найдите силу, которая его в движение. Она компенсирует силу трения , поэтому численно равна ей, но в сторону . Измерьте рулеткой или дальномером расстояние S, на которое сила F передвинула тело. Тогда работа силы трения будет равна произведению силы на расстояние со знаком «минус» A=-F∙S.

Пример. Автомобиль движется по дороге равномерно и прямолинейно. Какую работу сила трения на дистанции 200 м, если сила тяги двигателя равна 800 Н? При равномерном прямолинейном сила тяги двигателя равна по модулю силе трения . Тогда ее работа будет равна A=-F∙S =-800∙200=-160000 Дж или -160 кДж.

Если вы попробуете сдвинуть тяжелый шкаф, полный вещей, то как-то сразу станет понятно, что не так все просто, и что-то явно мешает благому делу наведения порядка.

  • И мешать движению будет не что иное, как работа силы трения , которую изучают в курсе физики седьмого класса.

С трением мы сталкиваемся на каждом шагу. В прямом смысле этого слова. Вернее было бы сказать, что без трения мы и шагу ступить не можем, так как именно силы трения удерживают наши ноги на поверхности.

Любой из нас знает, что такое ходить по очень скользкой поверхности — по льду, если этот процесс вообще можно назвать ходьбой. То есть, мы сразу видим очевидные плюсы силы трения. Однако, прежде чем говорить о пользе или вреде сил трения, рассмотрим для, начала, что такое сила трения в физике.

Сила трения в физике и ее виды

Взаимодействие, которое возникает в месте соприкосновения двух тел и препятствует их относительному движению, называют трением. А силу, которая характеризует это взаимодействие, называют силой трения.

  • Различают три вида трения: трение скольжения, трение покоя и трение качения.

Трение покоя

В нашем случае, когда мы пытались сдвинуть шкаф с места, мы пыхтели, толкали, краснели, но не сдвинули шкаф ни на дюйм. Что удерживает шкаф на месте? Сила трения покоя. Теперь другой пример: если мы положим руку на тетрадь и будем двигать ее по столу, то тетрадь будет двигаться вместе с нашей рукой, удерживаемая все той же силой трения покоя.

Трение покоя удерживает вбитые в стену гвозди, мешает самопроизвольно развязываться шнуркам, а также держит на месте наш шкаф, чтобы мы, случайно опершись на него плечом, не задавили любимого кота, который вдруг улегся подремать в тишине и покое между шкафом и стеной.

Трение скольжения

Вернемся к нашему пресловутому шкафу. Мы, наконец, сообразили, что сдвинуть его в одиночку нам не удастся и позвали на помощь соседа. В конце концов, исцарапав весь пол, вспотев, напугав кота, но, так и не выгрузив вещи из шкафа, мы передвинули его в другой угол.

Что мы обнаружили, кроме клубов пыли и не обклеенного обоями куска стены? Что, когда мы приложили силу, превышающую силу трения покоя, шкаф не просто сдвинулся с места, но и (с нашей помощью, естественно) продолжил двигаться дальше, до нужного нам места. И усилия, которые приходилось затрачивать на его передвижение, были примерно одинаковы на всем протяжении пути.

  • В данном случае нам мешала сила трения скольжения . Сила трения скольжения, как и сила трения покоя, направлена в сторону, противоположную приложенному воздействию.

Трение качения

В случае, когда тело не скользит по поверхности, а катится, то, возникающее в месте контакта трение, называют трением качения. Катящееся колесо немного вдавливается в дорогу, и перед ним образуется небольшой бугорок, который приходится преодолевать. Именно этим и обусловлено трение качения.

Чем тверже дорога, тем меньше трение качения. Именно поэтому ехать по шоссе намного легче, чем по песку. Трение качения в подавляющем большинстве случаев ощутимо меньше трения скольжения. Именно поэтому повсеместно применяют колеса, подшипники и так далее.

Причины возникновения сил трения

Первая — это шероховатость поверхности. Это хорошо понятно на примере досок пола или поверхности Земли. В случае же более гладких поверхностей, например, льда или покрытой металлическими листами крыши, шероховатости почти не видны, но это не значит, что их нет. Эти шероховатости и неровности цепляются друг за друга и мешают движению.

Вторая причина — это межмолекулярное притяжение, которое действует в местах контакта трущихся тел. Однако, вторая причина проявляется, в основном, лишь в случае очень хорошо отполированных тел. В основном же, мы имеем дело с первой причиной возникновения сил трения. И в таком случае, чтобы уменьшить силу трения, часто применяют смазку.

  • Слой смазки, чаще всего жидкий, разъединяет трущиеся поверхности, и трутся между собой слои жидкости, сила трения в которых в разы меньше.

Сочинение на тему «Сила трения»

В курсе физики седьмого класса школьникам дают задание написать сочинение на тему «Сила трения». Примером сочинения на эту тему может служить примерно такая фантазия:

«Допустим, решили мы на каникулах съездить к бабушке в гости на поезде. И не в курсе того, что как раз в это время вдруг, ни с того ни с сего, пропала сила трения. Проснулись, встаем с кровати и падаем, так как силы трения между полом и ногами нет.

Начинаем обуваться, и не можем завязать шнурки, которые не держатся из-за отсутствия силы трения. С лестницей вообще туго, лифт не работает — он уже давно лежит в подвале. Пересчитав копчиком абсолютно все ступени и доползя как-то до остановки, обнаруживаем новую беду: ни один автобус не остановился на остановке.

Чудом сели в поезд, думаем, какая красота — тут хорошо, топлива уходит меньше, так как потери на трение сведены к нулю, быстрее доедем. Но вот в чём беда: силы трения между колёсами и рельсами нету, а, значит, и оттолкнуться поезду не от чего! Так что, в общем, как-то и не судьба съездить к бабушке без силы трения.»

Польза и вред силы трения

Конечно же, это фантазия, и она полна лирических упрощений. В жизни все немного по-другому. Но, по сути, несмотря на то, что есть очевидные минусы силы трения, которые создают для нас ряд сложностей в жизни, очевидно, что без существования сил трения, проблем было бы куда как побольше. Так что нужно говорить, как о вреде сил трения, так и о пользе все тех же сил трения.

Примерами полезных сторон сил трения можно назвать то, что мы можем ходить по земле, что наша одежда не разваливается, так как нитки в ткани удерживаются благодаря все тем же силам трения, что насыпав на обледеневшую дорогу песок, мы улучшаем сцепление с дорогой, дабы избежать аварии.

Ну а вредом силы трения является проблема перемещения больших грузов, проблема изнашивания трущихся поверхностей, а также невозможность создания вечного двигателя, так как из-за трения любое движение рано или поздно останавливается, требуя постоянного стороннего воздействия.

Люди научились приспосабливаться и уменьшать, либо увеличивать силы трения , в зависимости от необходимости. Это и колеса, и смазка, и заточка, и многое другое. Примеров масса, и очевидно, что нельзя однозначно сказать: трение — это хорошо или плохо. Но оно есть, и наша задача — научиться использовать его на пользу человека.

Нужна помощь в учебе?

Предыдущая тема: Связь между силой тяжести и массой тела: динамометр.
Следующая тема:&nbsp&nbsp&nbspТрение в природе, быту и технике: еще больше ПРИМЕРОВ

183_notes:friction [Проекты и практики по физике]

Разделы 4.7 и 4.8 в Материи и взаимодействиях (4-е издание)

Контактные взаимодействия: нормальная сила и трение

Микроскопическая перспектива материалов помогает объяснить, как контактные взаимодействия происходят в природе. Контактные взаимодействия (силы) сами по себе не являются фундаментальными силами природы, но являются результатом электрических сил между атомами. Сжатие и растяжение материалов происходят не только на макроскопическом, но и на микроскопическом уровне. В этих заметках вы прочтете о том, как эти идеи порождают контактные силы, такие как нормальная сила и сухое трение.

Силы от контакта

Когда два объекта соприкасаются, их контактные поверхности действуют друг на друга. Это сильно отличается от гравитационной силы, потому что, хотя она действует на каждую часть массы, как вы узнаете, _мы считаем, что она действует в центре массы объекта. Это предположение не часто влияет на наши предсказания и объяснения движения. На самом деле, во всех моделях, которые вы использовали до сих пор, нас не интересовало, где именно сила действует на объект, а только то, что она действительно действует.

Для контактных сил (какое-то время) вы будете продолжать использовать предположение, что мы можем просто рассмотреть, действует контактная сила или нет (и в каком направлении она действует). В будущем вам может понадобиться точно знать, где он действует, потому что это может привести к тому, что движение будет немного сложнее. Например, вы можете опрокинуть коробку, если нажмете на нее в нужном месте, но ниже этого места она не опрокинется.

Нормальная сила

Мобильный телефон, помещенный на стойку, сжимает стол, поэтому нормальная сила может изменить его размер

Возможно, вы знаете о нормальной силе из школьного курса физики. Это плохое название, потому что оно не идентифицирует вещь, которая оказывает силу. Например, когда сотовый телефон лежит на столе, именно стол оказывает усилие, а не «нормальное». Слово «нормальный» подразумевает, что эта сила перпендикулярна поверхности, на которой находится объект.

Нормальная сила любопытна, потому что она может изменять свой размер (пока объект может выдерживать приложенную нагрузку). Сила, действующая на стол, может увеличиваться или уменьшаться при изменении нагрузки. Именно из-за электростатического взаимодействия между атомами в таблице. Нагрузка сжимает материал сильнее, если он тяжелее (рисунок справа). Рассмотрим размещение сотового телефона массой m на горизонтальном столе. Принцип импульса говорит вам, что результирующая сила, действующая на сотовый телефон, равна нулю (импульс не изменяется), и, следовательно:

$$\vec{F}_{net} = \vec{F}_{нормальный} + \vec{F}_{grav} = 0$$ $$\vec{F}_{normal} = — \vec{F}_{grav} = \langle 0, мг \rangle$$ $$F_{нормальный} = мг$$

Здесь вы можете выбрать любой сотовый телефон (или компьютер, или кирпич!) и пока изменение импульса остается нулевым, а стол может его выдержать, «нормальная» сила будет равна весу сотового телефона.

Видео-лекция

Трение

Начните толкать кирпич по столу, и атомы немного перестроятся; они сопротивляются движению

Трение – это сила сопротивления, возникающая из-за контакта двух тел. В то время как нормальная сила перпендикулярна контактным поверхностям, сила трения параллельна. Таким образом, векторная сумма этих двух сил (когда действуют обе) представляет собой силу, с которой поверхность действует на объект. То есть и нормальная сила, и сила трения обусловлены одними и теми же контактными взаимодействиями, их часто просто для удобства разделяют на параллельную и перпендикулярную поверхности составляющие.

Шероховатость двух соприкасающихся материалов показана на микроскопическом уровне.

Сила трения возникает из-за шероховатостей в материалах. Поверхность почти всех материалов шероховатая на микроскопическом уровне. Когда один объект перетаскивается через другой, эти шероховатые кусочки изгибаются, когда поверхности перемещаются, а затем изгибаются назад, когда они проходят друг мимо друга. Неровности выводятся из равновесия (импульс меняется), а затем возвращаются. В результате оба материала нагреваются; температура обоих объектов повышается. Как вы узнаете, это то, как трение рассеивает энергию.

Вы наверняка наблюдали это в своей повседневной жизни. Для перемещения объекта с постоянной скоростью часто требуется приложение постоянной силы. В физике вам говорят, что нет необходимости в силе, чтобы удерживать объект в движении. Трение помогает нам справиться с этим очевидным нарушением 1-го закона Ньютона. Если есть сила трения, противодействующая движению, то движение объекта замедлится, если только не будет приложена постоянная сила аналогичного размера в направлении, противоположном трению.

Трение может действовать в направлении движения

Подумайте о том, когда в следующий раз вы возьмете стакан. Сила трения между кончиками пальцев и стаканом направлена ​​в сторону движения, а именно поднесения стакана ко рту.

Модели трения

У нас есть две модели трения, каждая из которых имеет собственное математическое представление. Первое – трение скольжения, второе – трение покоя.

Трение скольжения

Когда объект скользит по поверхности, неровности постоянно изгибаются и возвращаются, рассеивая энергию и заставляя скользящий объект замедляться и нагреваться. Хотя связь между этим микроскопическим представлением о трении и макроскопическим представлением является предметом текущих исследований, у нас есть простая модель, которая хорошо работала в течение последних нескольких столетий. Здесь сила трения пропорциональна нормальной силе, а константа пропорциональности представляет собой экспериментально измеренную величину, называемую «коэффициентом кинетического трения».

$$F_{трение,скольжение} \ приблизительно \mu_k F_{нормальный}$$

Этот коэффициент кинетического трения ($\mu_k$) принимает значения от 0 до 1 в зависимости от двух соприкасающихся поверхностей, при этом значение, близкое к 1, означает большее трение (т.

Эта модель трения не зависит ни от скорости объекта, ни от площади объекта. Это приблизительная модель, а не фундаментальная сила, но это хорошая модель для большинства макроскопических ситуаций.

Статическое трение

Когда объект не движется, но определенно присутствует сила трения, возникает ситуация, когда присутствует статическое трение. Рассмотрим угловую погрузочную площадку, в которой ящик стоит на док-станции, но не двигается. Как и в случае со скольжением, у нас пока нет фундаментального понимания этих сил (потому что в них вовлечено так много атомов), но продуктивная модель похожа на модель трения скольжения.

$$F_{трение,статическое} \leq \mu_s F_{нормальное}$$

Сила трения покоя может изменяться, но не больше, чем нормальная сила, умноженная на коэффициент трения покоя ($\mu_s$). Эта константа также принимает значения от 0 до 1 в зависимости от двух поверхностей. Но бывает так, что коэффициент трения скольжения всегда меньше коэффициента статического трения для тех же двух материалов.

$$ \mu_k \le \mu_s$$

Если вы толкаете большой ящик, какое-то время ящик не будет двигаться, потому что приложенная сила не превышает $\mu_s F_{normal}$. Как только это произойдет, коробка начнет скользить (с трением), и усилие, которое необходимо приложить, уменьшится. Однако, если ваша приложенная сила станет меньше $\mu_k F_{normal}$, то ящик перестанет скользить, и вам придется приложить большую силу, чтобы снова заставить его двигаться. Это часто называют «прерывистым скольжением» и происходит потому, что максимальная статическая сила трения между двумя поверхностями больше, чем сила трения скольжения.

Примеры

  • удерживание_блока_против_стены

  • скольжение_до_остановки

Бесконтактное трение | Природа Физика

Трение бесконтактное

Скачать PDF

Скачать PDF

  • Опубликовано:
  • Марк Бьюкенен  

Физика природы том 3 , страница 827 (2007)Процитировать эту статью

  • 1336 доступов

  • 4 Цитаты

  • Сведения о показателях

Возможно, несколько физических понятий получили меньше внимания в связи с их важностью, чем трение. Изучая «эффективные» понятия статического трения и трения скольжения, студенты-механики легко могут принять наше незнание этого явления за глубокие знания. Что мы знаем, так это то, что макроскопическое трение часто чувствительно зависит от микроскопических молекулярных деталей. Например, мы могли бы наивно ожидать, что шероховатые поверхности будут демонстрировать большее трение, однако иногда оно выше между идеально гладкими и молекулярно чистыми поверхностями.

Что еще более удивительно, трение может существовать даже между телами, которые вообще не соприкасаются. Например, измерения силы, необходимой для перемещения алюминиевого наконечника сканирующего туннельного микроскопа по поверхности золота, оценивают силу трения, линейную по скорости, порядка 10 -11 кг с -1 . Многие другие эксперименты, в том числе недавняя работа с тонкими полимерными пленками, в целом подтверждают существование силы бесконтактного трения между поверхностями, разделенными очень небольшими расстояниями.

Что за этим стоит? Как и многие загадки физики, эта, по-видимому, имеет квантовое происхождение и тесно связана с другими, казалось бы, «аномальными» поведением тел, когда они находятся близко друг к другу.

Даже если два тела разделены, они все равно связаны…

По закону Стефана-Больцмана обычно ожидается тепловой поток между двумя черными телами при температурах Тл 1 и Тл 2 быть пропорциональным T 2 4 Т 1 4 . Для двух бесконечных плоских тел передаваемая теплота не зависит от расстояния между телами. Но этот результат зависит от свойств излучения в дальней зоне, и эксперименты показывают, что тепловой поток действительно отличается, если расстояние между телами меньше примерно длина волны теплового излучения, когда эффекты ближнего поля и туннелирование фотонов существенно влияют на течение. Эксперименты последнего десятилетия показали, что туннелирование фотонов может увеличить теплообмен между проводниками на несколько порядков даже при комнатной температуре.

Какое отношение это имеет к «бесконтактному» трению? Судя по всему, довольно много, так как оба эффекта связаны с явлениями ближнего поля. Даже если два тела разделены достаточно далеко, так что частицы не могут переходить от одного к другому, они все равно связаны посредством флуктуирующего электромагнитного поля и ван-дер-ваальсовых взаимодействий. Квантовые флуктуации, создающие временную плотность тока в одном теле, вызывают коррелированную плотность в другом, создавая мгновенное притяжение.

Эти флуктуирующие поля способствуют только что описанной аномальной передаче тепла, а также создают трение, если два тела находятся в относительном движении. Как изящно описано в недавнем обзоре (А. И. Волокитин и Б. Н. Дж. Перссон, Ред. Мод. физ. 79 , 1291–1329; 2007), эффект возникает из-за того, что движение вызывает задержку во времени между плотностями тока в двух телах. Это отставание во времени создает помимо силы притяжения между телами диссипативную поперечную силу, препятствующую их движению, фундаментальное трение квантового происхождения, существующее даже при абсолютном нуле.

Эти эффекты, по-видимому, больше, чем просто гипотетические, и влияют не только на теоретически или экспериментально «идеальные» поверхности. Недавние эксперименты обнаружили силы трения даже для электронов, движущихся внутри квантовых ям, и мы можем ожидать новых сюрпризов по мере совершенствования технологии создания атомарных и наноразмерных материалов. Понятие трения может войти в элементарную механику под маской предельной простоты, но нам еще многое предстоит узнать об одном из самых распространенных явлений науки.

Авторы

  1. Марк Бьюкенен

    Просмотр публикаций автора

    Вы также можете искать этого автора в PubMed Google Scholar

Права и разрешения

Перепечатка и разрешения

Об этой статье

Ньютоновская механика — Действует ли сила трения на тело на плоской поверхности без внешней силы?

$\begingroup$

Я знаю, что если объект находится под наклоном, возникает трение как реакция на составляющую массы объекта, параллельную поверхности. Я знаю, что если объект находится на плоской поверхности и к нему приложена внешняя горизонтальная сила, то в качестве реакции возникает сила трения.

А как насчет объекта, лежащего на плоской поверхности без внешней силы? Сила трения не «существует» или не применяется? Если бы мне нужно было нарисовать FBD, была бы это просто сила гравитации и нормальная силовая реакция?

  • ньютоновская механика
  • силы
  • трение
  • диаграмма свободного тела

$\endgroup$

1

$\begingroup$

Я знаю, что если объект находится на наклонной поверхности, возникает трение реакция на составляющую массы тела, параллельную поверхность.

Точнее сказать, что существует статическое трение, действующее вверх по плоскости, равное и противоположное компоненту силы тяжести, действующей на объект вниз по плоскости. Статическое трение препятствует движению объекта по плоскости. Но если составляющая силы тяжести, действующей вниз по плоскости, превышает максимально возможную силу трения покоя, объект будет скользить по плоскости. Тогда сила трения, действующая вверх по плоскости, становится кинетической силой трения. Если вы сделаете диаграмму свободного тела, как предложил @Eagle, вы увидите это.

Я знаю, что если объект находится на плоской поверхности и снаружи, приложена горизонтальная сила, в качестве реакции возникает сила трения.

Верно.

А как насчет объекта, лежащего на плоской поверхности без каких-либо внешних сила? Сила трения не «существует» или не применяется? Если бы я нарисуйте FBD, будет ли это просто гравитационная сила и нормальная сила реакция?

Сила трения существует только против приложенной внешней силы. Оно равно нулю, если приложенная сила равна нулю. По мере увеличения приложенной внешней силы сила трения покоя увеличивается на ту же величину, так что результирующая сила равна нулю, и между объектом и поверхностью нет относительного движения. Это продолжается до тех пор, пока не будет достигнута максимальная сила трения покоя, которая для плоской поверхности равна $f_{max}=µmg$, где $µ$ — коэффициент трения покоя. Затем объект скользит, и кинетическое трение берет верх.

Надеюсь, это поможет.

$\endgroup$

2

$\begingroup$

Трение — это саморегулирующаяся сила, противодействующая относительному контактному движению двух поверхностей. Если поверхность не идеально гладкая, всегда будет трение. Если блок находится на плоской поверхности, трение выравнивается до нуля, поскольку у блока нет тенденции тереться о плоскую поверхность. Если бы вы приложили хоть малейшую силу в горизонтальном направлении, сила трения постаралась бы ей противодействовать.

$\endgroup$

1

$\begingroup$

Рассмотрим тело на поверхности. То, что мы называем трением между телом и поверхностью, на самом деле является усредненным эффектом взаимодействия между бесчисленным количеством атомов в местах соприкосновения тела и поверхности. Когда тело неподвижно на поверхности, все эти индивидуальные взаимодействия между атомами компенсируют друг друга — они находятся в равновесии, и результирующая сила равна нулю. Таким образом, в целом на тело не действует чистая сила трения — если бы это было не так, трение заставляло бы тело двигаться.

Когда к телу приложена небольшая внешняя сила, оно немного сдвинется, и все миллионы крошечных сил несколько изменятся, некоторые из них будут возрастать, а некоторые ослабевать, пока снова не будет достигнуто равновесие. Если сила постепенно увеличивается, тело будет продолжать двигаться частично до точки, в которой равновесие больше не может поддерживаться, — после этой точки дальнейшее увеличение внешней силы будет ускорять тело по поверхности.

$\endgroup$

5

$\begingroup$

Если бы максимальная статическая сила трения была выражена в форме вектора , это было бы: $$ \large{\vec{F}_{max} = -\mu_{s}\cdot\|\vec{F}_n\| \cdot \шляпа{\textbf{u}}_{\perp F_n}} $$ здесь,

  • $\mu_{s}$ — коэффициент статического трения
  • $\vec{F}_n$ — нормальная сила
  • $\hat{\textbf{u}}_{\perp F_n}$ — единичный вектор приложенной внешней силы, перпендикулярный вектору нормальной силы

Итак, сила трения покоя отсутствует, если:

  • поверхности не имеют трения
  • на тело не действует нормальная сила (например, режим невесомости при свободном падении тел)
  • внешняя сила не приложена перпендикулярно нормальной силе

Кстати, если бы сила трения покоя существовала без приложения внешней силы, то тело, положенное на плоскую поверхность, начало бы двигаться с ускорением (второй закон Ньютона) без какой-либо видимой причины. Таким образом, это нарушило бы причинно-следственную связь.

Однако элементы поверхности, вызывающие силу трения, называемые неровностями, всегда существуют независимо от того, индуцируется сила трения или нет. Неровности:

В качестве бонусных баллов. Типичное землетрясение вызывается тем же механизмом трения, когда сила трения покоя уже не в состоянии удержать пару склеенных дрейфующих тектонических плит.

$\endgroup$

Твой ответ

Зарегистрируйтесь или войдите в систему

Зарегистрируйтесь с помощью Google

Зарегистрироваться через Facebook

Зарегистрируйтесь, используя адрес электронной почты и пароль

Опубликовать как гость

Электронная почта

Требуется, но никогда не отображается

Опубликовать как гость

Электронная почта

Требуется, но не отображается

Нажимая «Опубликовать свой ответ», вы соглашаетесь с нашими условиями обслуживания, политикой конфиденциальности и политикой использования файлов cookie

.

Что такое трение? | Физика Фургон

Категория Выберите категориюО фургоне физикиЭлектричество и магнитыВсе остальноеСвет и звукДвижение вещейНовая и захватывающая физикаСостояния вещества и энергииКосмосПод водой и в воздухе

Подкатегория

Поиск

Задайте вопрос

Последний ответ: 22.10.2007

Q:

Мне нужно написать, что такое трение и привести несколько примеров. Я знаю, что некоторые маленькие игрушечные машинки называются фрикционными. Вы можете помочь?
— Анна Кэмпбелл (9 лет)
Мехико, Мексика

A:

Анна,
Трение — это сила, уменьшающая движение между двумя поверхностями. [более общее обсуждение см. ниже/ mw] Трение возникает из-за того, что большинство поверхностей не идеально гладкие. Даже столешница, которая может показаться гладкой, имеет небольшие выпуклости, если вы посмотрите на нее в действительно хороший микроскоп. Когда две поверхности пытаются пройти мимо друг друга, эти маленькие неровности сталкиваются и замедляют движение поверхностей вниз, вызывая то, что мы называем трением. Чем шероховатее поверхность, тем больше на ней неровностей и тем больше на нее будет влиять трение. Некоторые примеры этого — скатывание деревянного блока по пандусу. Если вы скатываете деревянный брусок вниз по пандусу, он замедляется за счет трения. Если вы покроете блок наждачной бумагой (что сделает его более шероховатым), блок будет скользить медленнее, потому что трение замедляет его сильнее. Трение также увеличивается, если вы сильнее прижимаете поверхности друг к другу. Таким образом, полный чемодан будет иметь большее трение, противодействующее его движению, если вы попытаетесь скользить по полу, чем пустой. Других примеров трения очень много, потому что трение происходит каждый раз, когда вы перемещаете две соприкасающиеся поверхности. Попробуйте осмотреть свой дом и поэкспериментировать самостоятельно с шероховатыми и гладкими поверхностями.
Дан

(опубликовано 22.10.2007)

Дополнение №1: типы трения

Q:

Q существует ли более одного типа трения
— Милли Сайкс, (9 лет)
Лестер Англия

А:

Да. Трение — это всеобъемлющая фраза для любой силы, противодействующей относительному движению соприкасающихся объектов. Примеры включают «трение», создаваемое парашютом в воздухе, или силу, противодействующую скользящему движению кирпича по столу, и т. д. и т. п.

LeeH
ш. Майк В.

(опубликовано 25.01.2008)

Дополнение № 2: какова формула трения?

Q:

какова формула трения?
— Боб

А:

Нет никакой «Формулы». Существует несколько видов трения и несколько видов эмпирических правил, разработанных людьми для описания трения.
Мы описали эту проблему во многих предыдущих сообщениях на этот вопрос. Если вам нужна дополнительная информация, я предлагаю вам посетить:  

LeeH

(опубликовано 20. 10.2009)

Дополнение №3: скорость и трение

Вопрос:

Влияет ли скорость объекта на величину трения?
— Брэндон (18 лет)
Милуоки, Висконсин

A:

Существует особый тип трения между двумя твердыми поверхностями, скользящими друг относительно друга, при котором сила трения почти не зависит от относительной скорости. Часто это первый вид трения, который вводят в курс физики, поэтому студенты часто думают, что трение не зависит от скорости. Даже для этих скользящих поверхностей это всего лишь приближение. В тех же курсах часто преподается статическое трение между двумя нескользящими поверхностями. Он имеет более высокий коэффициент и, следовательно, может быть немного больше, чем трение скольжения. Для вещей, которые скользят очень-очень медленно, коэффициент трения скольжения приближается к коэффициенту статики.

Для объектов, медленно движущихся в жидкости, существует сила трения, прямо пропорциональная их относительной скорости. Это, вероятно, самый простой тип трения для описания и понимания, поэтому в некотором смысле жаль, что он не первый, которому учат. Более быстрое движение относительно жидкости приводит к более сложной зависимости скорости.

Майк В.

(опубликовано 09.01.2010)

Дополнение №4: трение и фундаментальные силы

Вопрос:

Если трение является силой, как оно связано с 5 фундаментальными силами: СИЛЬНОЙ, ЭЛЕКТРОМАГНИТНОЙ, СЛАБОЙ и ТЯЖЕСТИ?
— учитель

A:

Во-первых: я считаю 4 фундаментальные силы в вашем списке, если только вы не считаете электрические и магнитные отдельно. Однако если бы вы это сделали, вам пришлось бы включать магнитоподобную часть каждой из фундаментальных сил отдельно. Допустим, у вас есть 3 или 4 взаимодействия: гравитация, сильное ядерное взаимодействие и электрослабое взаимодействие*. (Вы можете считать электрослабое взаимодействие равным 1 или 2.)

Хорошо, а как насчет сил трения? Как правило, это силы, которые преобразуют кинетическую энергию крупномасштабных относительных движений в различные формы мелкомасштабной случайной тепловой энергии. Почти все известные нам видимые силы являются формами электромагнетизма, включая обычные силы трения.

Я думаю, что в больших масштабах гравитация играет роль в трении. Например, приливные напряжения на Земле и Луне являются частью пути преобразования крупномасштабной орбитальной и вращательной энергии в тепловую. Однако последний этап, на котором энергия больших приливов преобразуется, например, в немного более теплые океаны, всегда является электромагнитным. Это потому, что в масштабе молекул гравитация ничтожна по сравнению с электромагнитными силами.

Майк В.

* Теперь мы должны также добавить Хиггса.

(опубликовано 08.03.2011)

Дополнение №5: Что, если бы не было трения?

Q:

Что произойдет, если не будет трения? Можете ли вы привести несколько примеров?
— Лейганн (14 лет)
NJ

A:

Если бы вы попытались идти, вы бы упали на задницу; учтите очень, очень скользкий лед. Если вы попытаетесь водить машину, во-первых, вы не сможете ее завести, а во-вторых, если вам удастся завести ее, вы не сможете остановиться. Все эти процессы связаны с трением того или иного рода.

LeeH

(опубликовано 24.10.2011)

Дополнение №6: виды трения

Вопрос:

я хочу знать, сколько существует видов трения
— k.suresh, лектор (25 лет)
hyderabad,ap,india

A:

Трение возникает во многих ситуациях по-разному, а не только из-за неровных поверхностей, о которых говорил Дэн. Например, существует тип трения между магнитом и движущимся электрическим проводником, вызванный вихревыми токами, хотя магнит и проводник не соприкасаются. Некоторые старомодные проигрыватели фонографов использовали это для регулировки скорости путем перемещения магнита. Существует трение между любым объектом и жидкостью (жидкостью или газом), в которой он движется. Я не думаю, что будет очень полезно пытаться составить список определенного числа типов трения, потому что категории часто пересекаются.

Mike W.

(опубликовано 16.05.2013)

Последующее наблюдение по этому ответу

Связанные вопросы

  • Расчет коэффициента фрикма

  • FRICTION и Falling Saturd

    444449
  • FRICTION и Falling Plaging

    44444449
  • FRICTION.

    быстрое торможение автомобиля

  • нагрев бурового долота

  • работа = сила, умноженная на расстояние?

  • почему воздушные шары не двигаются боком?

  • вращение под действием силы тяжести

  • Гонка перьев и пушечных ядер в различных газах

  • песчаные шары катятся вниз

  • гравитация, сопротивление воздуха, падение

  • 9000 Все еще любопытно?

    Вопросы и ответы по Expore в смежных категориях

    • Трение

    Силы трения « KaiserScience

    Вот пингвин с низким коэффициентом трения 🙂

    Попробуйте продвинуть тяжелый предмет по полу:


    сначала может не двигаться.
    Трение удерживает объект на месте:
    объект статичен (не движется).

    Когда вы прикладываете достаточную силу, объект начинает двигаться:


    он кинетический (в движении)
    Теперь нам нужно прикладывать меньше силы, чтобы поддерживать движение .

    Вывод 1: трение покоя — большая сила.


    Сначала, чем больше силы вы прикладываете, тем сильнее отталкивает трение.
    Удерживает объект на месте.

    Вывод 2: Как только вы преодолеете определенный барьер,


    большая часть сил трения нарушена:
    Что бы ни вызывало силы статического трения,
    нельзя восстановить во время движения объекта.
    Таким образом, при движении трение всегда будет намного меньше.

    Посмотрим результаты эксперимента — брусок на шероховатой поверхности.

    Прикрепите к нему пружинную шкалу – и потяните блок.

    Шкала позволяет нам увидеть, какую силу мы прикладываем к блоку, когда тянем его.

    По мере того, как мы тянем все больше и больше, мы видим увеличение силы, но блок не двигается (он статичен)

    Внезапно блок вырывается и начинает двигаться – и в этот момент сила на весах падает!

    Пока блок движется, сила меньше, чем была, когда он стоял на месте.

    Это визуально показывает, что  [статический коэффициент трения] >[кинетический коэффициент трения]

    http://hyperphysics.phy-astr.gsu.edu/hbase/frict2.html

    Эксперимент: добавьте массу к объекту. Вам нужно будет сильнее нажимать (или тянуть), чтобы начать, а также больше нажимать или тянуть, чтобы он продолжал двигаться.

    Эксперимент: Если вы смазали пол маслом, то объект будет двигаться легче. Вывод: некоторые жидкости могут нарушить образование всего, что создает трение.

    Вот кошка — обратите внимание, что коэффициент трения между лапами кошки и столом очень низкий.

    Каково молекулярное происхождение трения?

    Поверхности шероховатые. Когда вы заставляете объект двигаться, вы должны (слегка) приподнять объект, пока он не сможет прыгать вперед — только кончики поверхности ударяются или ломаются.

    http://www.drcruzan.com/Friction.html

    Обратите внимание, что трение всегда препятствует движению.

    Сила трения пропорциональна силе сжатия

    http://www.drcruzan.com/Friction.html

    Чем сильнее прижимаются поверхности друг к другу (гравитация), тем больше силы требуется для их перемещения.

    Источник трения?

    * сила сцепления между поверхностными молекулами двух объектов:

    * адгезия зависит от типа поверхности

    * это сложный аспект физики поверхности.

    Когда объект движется, становится меньше точек соприкосновения, поэтому требуется меньшее усилие, чтобы поддерживать его движение.

    * При малых скоростях трение практически не зависит от скорости.

    (текст выше адаптирован из ABE Advanced Level Physics, полученного из College Physics от OpenStax College) молекулярное происхождение трения

    http://www.drcruzan.com/Friction.html

    http://archive.cnx.org/contents/904e502e-c9ec-4759-b7d1-3827fd3ab274@1/5-2-friction

    https: //phet.colorado.edu/en/simulation/friction

    https://www. quora.com/At-an-atomic-level-what-is-the-difference-between-static-and-kinetic-friction

    Выдержка из:

    http://www.mrwaynesclass.com/freebodies/reading/index01.html

    * много сил трения.

    * точка, противоположная направлению движения и параллельная поверхностям контакта.

    * В некоторых книгах этот символ пишется курсивом «F», но мы будем использовать «f».

    * ориентируемся на трение из-за контакта между двумя поверхностями.

    * мы используем греческую букву

    мю мк для коэффициента трения.

    http://hyperphysics.phy-astr.gsu.edu/hbase/frict3.html

    Коэффициент трения в процентах от

    нормальной силы .

    * найдено экспериментальным путем


    * зависит от природы контактирующих материалов.
    * Иногда трение отсутствует (например, на скользком льду)

    Материал верха

    Материал дна

    мк

    Статический коэффициент трения

    мк

    Кинетический коэффициент трения

    лед

    лед

    0,02

    – – –

    дерево

    на сухой древесине

    0,3

    0,2

    железо

    на чистом железе

    1,1

    0,15

    железо

    на чистом, сухом дубе

    – – –

    0,49

    графит

    по стали

    0,1

    кожа

    на чистом стекле

    0,9 – 1,0

    – – –

    Резиновая шина

    на асфальте

    0,7

    0,45

    Источники:
    http://hyperphysics. phy-astr.gsu.edu/Hbase/Mechanics/frictire.html
    http://physics.info/friction
    http://www.engineershandbook.com/Tables/frictioncoefficients.htm

    Статическое трение = две поверхности не скользят друг по другу.

    Кинетическое трение = две поверхности скользят друг по другу.

    http://www.mrwaynesclass.com/freebodies/reading/index01.html

    Девушка, стоящая на горке (справа), испытывает статическое трение между ее туфлями и горкой.


    Эти две поверхности не двигаются относительно друг друга.

    Девушка, движущаяся вниз (слева), испытывает кинетическое трение между штанами и горкой.


    Это связано с тем, что штаны двигаются относительно поверхности слайда.

    В обоих случаях трение направлено против направления движения и параллельно поверхности.


    Свободное тело каждой девушки будет выглядеть так, как показано ниже.
    (другие силы затемнены, чтобы выделить трение)

    http://www.mrwaynesclass.com/freebodies/reading/index01.html

    Трение действует параллельно соприкасающимся поверхностям и в направлении, противоположном движению объекта.

    Трение зависит от природы материалов и их гладкости.

    Кинетическое (скольжение) трение меньше статического трения.

    Кинетическое трение практически не зависит от скорости.

    Трение практически не зависит от площади контакта. (это не интуитивно)

    Трение пропорционально силе, прижимающей поверхности друг к другу.

    Нанесение силы трения на диаграмму свободного тела.

    Обратите внимание, что вектор силы трения (1) параллелен силе, (2) противоположен направлению движения или предполагаемому движению.

    Напряжение

         Напряжение существует в любом теле, которое притягивается противодействующими силами.

    Обычно мы говорим о веревках и цепях как о натянутых, но любое тело может быть растянуто.

     ================================================ ====

    Брусок массой m = 0,75 кг тянут по столу с постоянной скоростью. Если для поддержания постоянной скорости требуется сила 2,0 Н, рассчитайте коэффициент трения для этой системы.

    Решение: Сначала нарисуйте схему ситуации. Масса 0,75 кг действует на стол с силой (F = мг), и нормальная сила равна ей. Я не удосужился сделать эти две силы противоположными по знаку, но, строго говоря, это силы с противоположными векторами. В этих случаях постоянной скорости ускорение отсутствует, поэтому все силы должны быть уравновешены. Поэтому не имеет значения, как быстро движется блок, важно только то, что он движется с постоянной скоростью. Тогда сила трения Ff равна силе тяги 2,0 Н (но опять же противоположного знака, который я не буду учитывать).

    http://www.drcruzan.com/Friction.html

    Теперь достаточно изменить уравнение трения и использовать нормальную силу и силу трения для расчета коэффициента трения. Обратите внимание, что при вычислении μ единицы измерения сокращаются.

    http://www.drcruzan.com/Friction.html

    Пример 2

    Груз массой 4,5 кг поднимается по 30-градусной рампе с постоянной скоростью под действием падающего груза массой 8,0 кг, как показано на рисунке. Вычислите коэффициент трения.

    Решение: Вот схема.

    http://www.drcruzan.com/Friction.html

     Все соответствующие расчеты показаны выше. Обратите внимание, что это еще одна из тех проблем, которые упрощают поиск µ, потому что сила на рампе (22,0 Н) равна силе трения. Нормальная сила 38,2 Н дает результат:

     

    Стандарты обучения

    2016 Массачусетская учебная программа по науке и технике/инженерному делу

    3-ПС2-1. Приведите доказательства, объясняющие влияние нескольких сил, включая трение, на объект
    . Включите уравновешенные силы, которые не изменяют движение объекта, и
    неуравновешенных сил, которые изменяют движение объекта.

    ГС-ПС2-1. Проанализируйте данные, подтверждающие утверждение о том, что второй закон Ньютона представляет собой математическую модель 90 336, описывающую изменение движения (ускорение) объектов, когда на 90 336 действует результирующая сила… Силы могут включать контактные силы, в том числе трение.

    СТРУКТУРА НАУЧНОГО ОБРАЗОВАНИЯ K-12: Практика, сквозные концепции и основные идеи
    PS2.B: ТИПЫ ВЗАИМОДЕЙСТВИЙ
    Какие основные силы объясняют разнообразие наблюдаемых взаимодействий?

    Все силы между объектами возникают в результате нескольких типов взаимодействий… Любые два объекта, находящиеся в контакте, также воздействуют друг на друга силами электромагнитного происхождения. Эти силы возникают в результате деформаций подструктур объектов и электрических зарядов частиц, образующих эти подструктуры (например, стол, поддерживающий книгу, силы трения).

    Массачусетс Структура учебного плана по науке и технике/инженерному делу (2006 г. )

    1. Движение и силы. Центральная концепция: законы движения и гравитации Ньютона описывают и предсказывают движение большинства объектов.
    1.6 Качественно различать статическое и кинетическое трение и описывать их влияние на движение объектов.

    Нравится:

    Нравится Загрузка…

    Сила и трение — веб-формулы

    Сила: 909:25 Сила определяется как изменение величины скорости объекта или изменение направления его движения. Сила может изменить форму и размер объектов.

    Сила, которая не изменяет состояние объекта, называется уравновешенной силой. Сила толкания и сила трения, действующие на объект, уравновешены, и на него не действует результирующая внешняя сила, поэтому объект движется с постоянной скоростью.

    Сила трения – это сила, возникающая между двумя соприкасающимися поверхностями и направленная всегда в направлении, противоположном давлению.

    Сила, действующая на объект и приводящая его в движение, называется неуравновешенной силой. Неуравновешенная сила вызывает изменение скорости объекта или изменение направления объекта. Если неуравновешенная сила полностью устранена, объект будет продолжать двигаться с той скоростью, которую он приобрел до этого момента.

    Движение объекта впервые было описано Галилеем на примере катания шариков по наклонной плоскости. Он пришел к выводу, что тела движутся с постоянной скоростью, когда на них не действует никакая сила, скорость тела увеличивается, когда оно скатывается по наклонной плоскости, и уменьшается, когда оно поднимается вверх.

    Ньютон изучил идеи Галилея о силе и движении и представил три фундаментальных закона движения. Эти три закона известны как законы движения Ньютона.

    Закон движения Ньютона:
    Первый закон движения формулируется так: объект остается в состоянии покоя или равномерного прямолинейного движения, если только к нему не принуждают изменить это состояние под действием приложенной силы.

    Первый закон движения:
    Все объекты сопротивляются изменению своего состояния движения. Они имеют тенденцию оставаться в покое или продолжать двигаться с той же скоростью. Это называется инерцией, а первый закон движения также известен как закон инерции.

    Инерция определяется как тенденция объекта оставаться в покое или двигаться до тех пор, пока на него не подействует внешняя сила. Объект с большей массой имеет большую инерцию. Масса объекта является мерой его инерции.

    Первый закон движения указывает, что когда на объект действует неуравновешенная внешняя сила, его скорость изменяется и увеличивается.
    Ньютон ввел свойство, объединяющее массу объекта и его скорость, и назвал его импульсом.

    Импульс (p) объекта определяется как произведение его массы (m) и скорости (v).

      p = mv

    Единица импульса в системе СИ – килограмм-метр в секунду или кг-м/с
    Импульс имеет как направление, так и величину. Изменение импульса определяется величиной силы и скоростью изменения импульса.

    Второй закон движения:
    Второй закон движения гласит, что скорость изменения количества движения объекта пропорциональна приложенной неуравновешенной силе в направлении силы.

    Второй закон движения измеряет действие силы на объект как произведение его массы и ускорения.

    Предположим, что объект массой (m) движется прямолинейно с начальной скоростью (u). Он равномерно ускоряется до скорости (v) за время (t) за счет приложения постоянной силы (F) в течение всего времени (t). Конечный и начальный импульс объекта будет равен
    р 1 = мю
    p 2 = mv

    Изменение импульса p ∞ p 2 1 ∞ мв-мю ∞ м (в-у)
    Скорость изменения количества движения ∞ м (v-u)/t
    Или приложенная сила, F ∞ м (v-u)/t
    F = К м (v-u)/t
    F = K m a   (a = v-u/t — ускорение)
    К = константа
    F =
    млн лет СИ единица силы = кг м с 2 или Ньютон

    Первый закон движения можно математически сформулировать из математического выражения для второго закона движения.
    F = ма 909:25 F = m [(v-u)/t]
    Ft = mv-mu
    Теперь, если F = 0, v = u для любого времени, то объект будет продолжать двигаться с постоянной скоростью (u) в течение всего времени (t).
    Если u равно нулю, то v также будет равно нулю. То есть объект останется в покое.

    Третий закон движения:
    Третий закон движения гласит, что когда один объект воздействует на другой объект, второй объект немедленно оказывает обратное воздействие на первый объект. Эти две силы всегда равны по степени, но противоположны по направлению. Эти две противоположные силы также известны как силы действия и противодействия, а закон известен как третий закон Ньютона или законы действия и противодействия. 909:25 Силы действия и противодействия всегда равны по величине, но не могут вызывать одинаковое ускорение, потому что каждая сила действует на разные массы и разные объекты.

    Расчеты:
    Пример-1: Сила трения всегда противоположна ………..
    а) боевая группа
    б) Уравновешенная сила
    в)  Толкающая сила
    d)  Нет

    Ответ: Сила трения – это сила, возникающая между двумя соприкасающимися поверхностями и направленная всегда в направлении, противоположном давлению.

alexxlab

Добавить комментарий

Ваш адрес email не будет опубликован. Обязательные поля помечены *